Крок 1 - Медицина 2015 (буклет)

1 / 200
В шкірі виявлена щільна, рухома, чітко відмежована від оточуючих тканин пухлина. На розрізі вона білого кольору, представлена волокнистою тканиною. Мікроскопічно: хаотично переплетені колагенові волокна, клітин мало. Що це за пухлина? A dense, mobile tumor, clearly separated from the surrounding tissues, was detected in the skin. On cross-section, it is white in color, represented by fibrous tissue. Microscopically: chaotically intertwined collagen fibers, few cells . What kind of tumor is this?

Міома Myoma

Гістіоцитома Histiocytoma

Дерматофіброма Dermatofibroma

Фіброма Fibroma

Десмоїд Desmoid

2 / 200
Для корекції артеріального тиску при колаптоїдному стані хворому було введено мезатон. Який механізм дії даного препарату? Mezaton was administered to the patient to correct blood pressure in a colaptoid condition. What is the mechanism of action of this drug?

Блокує β-адренорецептори Blocks β-adrenoceptors

Стимулює β-адренорецептори Stimulates β-adrenoceptors

Стимулює α-адренорецептори Stimulates α-adrenoceptors

Блокує α-адренорецептори Blocks α-adrenoceptors

Стимулює α- і β-адренорецептори Stimulates α- and β-adrenoceptors

3 / 200
Після переходу до змішаного харчування у новонародженої дитини виникла диспепсія з діареєю, метеоризмом, відставанням у розвитку. Біохімічна основа даної патології полягає у недостатності: After switching to a mixed diet, the newborn child developed dyspepsia with diarrhea, flatulence, and developmental delay. The biochemical basis of this pathology is a deficiency:

Сахарази та ізомальтази Sucrases and isomaltases

Ліпази та креатинкінази Lipases and creatine kinases

Трипсину та хімотрипсину Trypsin and chymotrypsin

Целюлази Cellulases

Лактази та целобіази Lactases and cellobiases

4 / 200
У чоловіка 52-х років діагностовано системний амебіаз з ураженням кишківника, печінки, легенів. Який препарат слід призначити? A 52-year-old man was diagnosed with systemic amoebiasis with damage to the intestines, liver, and lungs. What drug should be prescribed?

Хінгамін Hingamine

Ентеросептол Enteroseptol

Тетрациклін Tetracycline

Хініофон Hineophone

Метронідазол Metronidazole

5 / 200
У пацієнта з ознаками коліту виділена чиста культура бактерій, яка за морфологічними, культуральними та біохімічними властивостями належить до роду шигел. Яку з названих реакцій доцільно застосувати для серологічної ідентифікації культури? A pure culture of bacteria was isolated from a patient with signs of colitis, which according to morphological, cultural and biochemical properties belongs to the genus Shigella. Which of the above reactions should be used for serological identification of the culture ?

Затримки гемаглютинації Hemagglutination delays

Аглютинації з діагностичними сироватками Agglutinations with diagnostic sera

Непрямої гемаглютинації Indirect hemagglutination

Преципітації Precipitation

Зв’язування комплементу Complement binding

6 / 200
Для лікування урогенітальних інфекцій використовують хінолони - інгібітори ферменту ДНК-гірази. Який процес порушується під дією хінолонів у першу чергу? Quinolones are used to treat urogenital infections - inhibitors of the DNA gyrase enzyme. What process is disrupted by quinolones in the first place?

Репарація ДНК DNA Repair

Ампліфікація генів Gene Amplification

Рекомбінація генів Gene recombination

Зворотна транскрипція Reverse transcription

Реплікація ДНК DNA Replication

7 / 200
У хворого з аневризмою правої підключичної артерії спостерігається захриплість голосу. З подразненням якого нерву це може бути пов’язано? A patient with an aneurysm of the right subclavian artery has a hoarse voice. Which nerve could this be due to irritation?

N.laringeus superior dexter N.laryngeus superior dexter

N.laringeus superior sinister N.laryngeus superior sinister

N.laringeus inferior sinister N.laryngeus inferior sinister

N.laringeus reccurens sinister N.laryngeus reccurens sinister

N.laringeus reccurens dexter N.laryngeus reccurens dexter

8 / 200
Під час операції в печінці хворого виявлені дрібні пухирці малих розмірів з незначною кількістю рідини, які щільно прилягають один до одного. Який гельмінтоз виявився у хворого? During the operation in the patient's liver, small blisters of small sizes with a small amount of liquid were found, which are closely adjacent to each other. What kind of helminthiasis was found in the patient?

Альвеококоз Alveococcosis

Клонорхоз Clonorchosis

Фасціольоз Fasciolosis

Опісторхоз Opistorchosis

Дікроцеліоз Dicroceliosis

9 / 200
У хлопчика 3-х років з вираженим геморагічним синдромом відсутній антигемофільний глобулін А (фактор VIII) у плазмі крові. Яка фаза гемостазу первинно порушена у цього хворого? A 3-year-old boy with severe hemorrhagic syndrome has no antihemophilic globulin A (factor VIII) in his blood plasma. Which phase of hemostasis is primarily disturbed in this patient?

Перетворення фібриногену в фібрин Conversion of fibrinogen to fibrin

Перетворення протромбіну в тромбін Conversion of prothrombin to thrombin

Зовнішній механізм активації протромбінази External mechanism of prothrombinase activation

Внутрішній механізм активації протромбінази Internal mechanism of prothrombinase activation

Ретракція кров’яного згустку Blood clot retraction

10 / 200
До лікаря звернувся чоловік 27-ми років. При огляді було виявлено збільшення кистей, стоп та нижньої щелепи. Крім того спостерігалися деформація суглобів (kiphosis), гормональні порушення (імпотенція, атрофія яєчок). Функція якої залози порушена? A 27-year-old man consulted a doctor. An examination revealed an increase in the hands, feet, and lower jaw. In addition, joint deformation (kyphosis), hormonal disorders ( impotence, testicular atrophy). Which gland's function is impaired?

Прищитоподібні залози Parathyroid glands

Передня частка гіпофізу Anterior pituitary lobe

Шишкоподібне тіло Cones

Надниркові залози Adrenal glands

Щитоподібна залоза Thyroid

11 / 200
У чоловіка 25-ти років діагностований гострий дифузний гломерулонефрит. З анамнезу: за 18 днів до прояву хвороби переніс ангіну. Який механізм ураження ниркових клубочків буде спостерігатися у цьому випадку? A 25-year-old man was diagnosed with acute diffuse glomerulonephritis. From the anamnesis: 18 days before the onset of the disease, he had angina. What mechanism of kidney glomeruli damage will be observed in this case ?

- -

Нефротоксичний Nephrotoxic

Медикаментозний Medical

Ішемічний Ischemic

Імунний Immune

12 / 200
У чоловіка 35-ти років феохромоцитома. В крові спостерігається підвищений рівень адреналіну та норадреналіну, концентрація вільних жирних кислот зросла в 11 разів. Активація якого ферменту під впливом адреналіну підвищує ліполіз? A 35-year-old man has a pheochromocytoma. There is an elevated level of adrenaline and noradrenaline in the blood, the concentration of free fatty acids has increased 11 times. The activation of which enzyme under the influence of adrenaline increases lipolysis?

Фосфоліпаза С Phospholipase C

Фосфоліпаза А2 Phospholipase A2

Ліпопротеїдліпаза Lipoprotein lipase

Холестеролестераза Cholesterolesterase

ТАГ -ліпаза TAG lipase

13 / 200
Жінку 44-х років втяла оса, внаслідок чого розвинувся шок. В анамнезі - тяжка алергічна реакція на укус оси. Об’єктивно: РS- 179/хв, слабкий, АТ- 80/40 мм рт.ст., ЧД-26/хв. Яка провідна ланка патогенезу анафілактичного шоку? A 44-year-old woman was stung by a wasp, as a result of which she developed shock. She has a history of a severe allergic reaction to a wasp sting. Objectively: RS- 179/min, weak, blood pressure - 80/40 mm Hg, blood pressure - 26/min. What is the leading link in the pathogenesis of anaphylactic shock?

Зниження периферійного опору судин Reduction of peripheral vascular resistance

Зменшення об’єму циркулюючої крові Decreasing the volume of circulating blood

Зменшення ударного об’єму серця Decreased stroke volume of the heart

Тахікардія Tachycardia

Біль Pain

14 / 200
Фекалії дитини, що хворіє на ентерит, емульгують в фізіологічному розчині і краплю емульсії наносять на елективне середовище: 10% молочно-сольовий, або жовтковосольовий агар. Який мікроорганізм передбачається виділити? Feces of a child suffering from enteritis are emulsified in a physiological solution and a drop of the emulsion is applied to an elective medium: 10% milk-salt or yolk-salt agar. What microorganism is expected highlight?

Стрептокок Streptococcus

Кишкова паличка Escherichia coli

Стафілокок Staphylococcus

Ентерокок Enterococcus

Клебсієла Klebsiella

15 / 200
У хворої 63-х років діагностований інсулінонезалежний цукровий діабет. Ендокринолог почав лікування з призначення глібенкламіду. Вкажіть механізм дії цього засобу: A 63-year-old patient was diagnosed with non-insulin-dependent diabetes. The endocrinologist started treatment by prescribing glibenclamide. Specify the mechanism of action of this agent:

Зменшує всмоктування глюкози в кишківнику Reduces absorption of glucose in the intestine

Активує в-клітини острівців Лангерганса Activates b-cells of the islets of Langerhans

Гальмує транспорт глюкози до клітин Inhibits glucose transport to cells

Стимулює гіпоталамічні центри Stimulates hypothalamic centers

Підсилює руйнування білків Strengthens the destruction of proteins

16 / 200
У хворого коса пахвинна грижа. Яке анатомічне утворення стало слабким місцем передньої черевної стінки? The patient has an oblique inguinal hernia. What anatomical formation became the weak point of the anterior abdominal wall?

Медіальна пахвинна ямка Medial inguinal fossa

Надміхурова ямка Scrotal fossa

Стегнова ямка Femoral fossa

Латеральна пахвинна ямка Lateral inguinal fossa

Пахвинний трикутник Inguinal triangle

17 / 200
Людині внутрішньовенно ввели 0,5 л ізотонічного розчину лікарської речовини. Які з рецепторів насамперед відреагують на зміну водно-сольового балансу організму? A person intravenously injected 0.5 l of an isotonic solution of a medicinal substance. Which of the receptors will first of all react to a change in the body's water-salt balance?

Барорецептори дуги аорти Baroreceptors of the aortic arch

Осморецептори гіпоталамусу Osmoreceptors of the hypothalamus

Натрієві рецептори гіпоталамусу Sodium receptors of the hypothalamus

Осморецептори печінки Osmoreceptors of the liver

Волюморецептори порожнистих вен і передсердь Volume receptors of the vena cava and atria

18 / 200
До лікарні надійшла дитина з діагнозом 'стафілококовий сепсис'. На яке живильне середовище потрібно посіяти кров хворого з метою виділення збудника? A child was admitted to the hospital with a diagnosis of 'staphylococcal sepsis'. On which nutrient medium should the patient's blood be inoculated in order to isolate the pathogen?

Цукрово-пептонний бульйон Sugar-peptone broth

Середовище Бучіна Environment of Buchin

Жовчно-сольовий агар Bile-salt agar

М ’ясо-пептонний агар Meat-peptone agar

Середовище Плоскірева Environment of Ploskirev

19 / 200
При бактеріоскопічному дослідженні носоглоткового слизу дитини 2,5 років, хворої на назофарингіт, виявлені грамнегативні диплококи, схожі за формою на кавові зерна. Які структури організму дитини найбільш імовірно будуть уражені, якщо ці мікроорганізми проникнуть у кров? During the bacterioscopic examination of the nasopharyngeal mucus of a 2.5-year-old child with nasopharyngitis, gram-negative diplococci were found, similar in shape to coffee beans. What structures of the child's body are most likely to be affected if these microorganisms enter the blood?

Серцеві клапани Heart valves

Ниркові гломерули Kidney glomeruli

Оболонки мозку Meninges

Сечостатеві шляхи Urinogenital tract

Лімфатичні вузли Lymph nodes

20 / 200
У хлопчика 2-х років спостерігається збільшення в розмірах печінки та селезінки, катаракта. В крові підвищена концентрація цукру, але тест толерантності до глюкози в нормі. Спадкове порушення обміну якої речовини є причиною цього стану? A 2-year-old boy has an increase in the size of the liver and spleen, a cataract. The concentration of sugar in the blood is increased, but the glucose tolerance test is normal. Hereditary metabolic disorder what substance is the cause of this condition?

Галактоза Galactose

Сахароза Sucrose

Глюкоза Glucose

Мальтоза Maltose

Фруктоза Fructose

21 / 200
В сечі новонародженого визначається цитрулін та високий рівень аміаку. Вкажіть, утворення якої речовини, найімовірніше, порушене у цього малюка: Citrulline and a high level of ammonia are detected in the urine of a newborn. Indicate the formation of which substance is most likely impaired in this baby:

Сечова кислота Uric acid

Сечовина Urea

Аміак Ammonia

Креатинін Creatinine

Креатин Creatine

22 / 200
У хворого, що звернувся до лікарню зі скаргами на пронос, діагностували амебну дизентерію. До комплексного лікування був включений доксациклін. Назвіть вид дії призначеного препарату: A patient who came to the hospital with complaints of diarrhea was diagnosed with amoebic dysentery. Doxacycline was included in the complex treatment. Name the type of action of the prescribed drug:

Рефлекторна дія Reflex action

Основна дія Main Action

Етіотропна дія Etiotropic action

Незворотня дія Irreversible action

Пряма дія Direct Action

23 / 200
Під час обстеження у хворої встановлене ураження дорсальної частини мосту, порушена функція жування. Ядро якого нерва уражене? During the examination, the patient was diagnosed with a lesion of the dorsal part of the pons, chewing function was impaired. What nerve nucleus is affected?

Рухове ядро трійчастого нерва Motor nucleus of the trigeminal nerve

Ядро під’язикового нерва Nucleus of the hypoglossal nerve

Подвійне ядро блукаючого нерва Dual nucleus of the vagus nerve

Рухове ядро лицевого нерва Motor nucleus of the facial nerve

Мостове ядро трійчастого нерва Principal trigeminal nucleus

24 / 200
з дихальною недостатністю рН крові 7,35. Визначення рСO2 24. показало наявність гіперкапнії. При дослідженні рН сечі відзначається підвищення її кислотності. Яка форма порушення кислотноосновного стану в даному випадку? with respiratory insufficiency blood pH 7.35. The determination of pCO2 24. showed the presence of hypercapnia. When examining the pH of urine, its acidity is noted to increase. What is the form of violation of the acid-base state in this case?

Алкалоз газовий, компенсований Gas alkalosis, compensated

Ацидоз метаболічний, компенсований Metabolic acidosis, compensated

Ацидоз газовий, компенсований Gas acidosis, compensated

Ацидоз метаболічний, декомпенсований Metabolic acidosis, decompensated

Алкалоз газовий, декомпенсований Gas alkalosis, decompensated

25 / 200
При бактеріологічному дослідженні сечі хворого на пієлонефрит виділені мікроорганізми, що утворюють на м’ясо-пептонному агарі жовто-зелений пігмент і мають характерний запах. Як вони називаються? During a bacteriological examination of the urine of a patient with pyelonephritis, microorganisms were isolated that form a yellow-green pigment on meat-peptone agar and have a characteristic odor. What are they called?

Клебсієли Klebsiels

Ешеріхії Escherichia

Азотобактерії Azotobacteria

Псевдомонади Pseudomonads

Протеї Proteus

26 / 200
У хворого впродовж 10-ти днів має місце підвищена температура, напади характерного кашлю. Лікар призначив посів слизу з носоглотки на середовище КВА. Який мікроорганізм передбачається виявити? The patient has had a high temperature for 10 days, attacks of a characteristic cough. The doctor prescribed a culture of mucus from the nasopharynx on KVA medium. What microorganism is expected to be detected?

Палочка інфлуенци Influenza

Паличка коклюшу Whooping cough

Лістерія Listeria

Стафілокок Staphylococcus

Клебсієла Klebsiella

27 / 200
У нейрохірургічне відділення надійшов 54-річний чоловік зі скаргами на відсутність чутливості шкіри нижньої повіки, латеральної поверхні носа, верхньої губи. Лікар при огляді встановив запалення другої гілки трійчастого нерва. Через який отвір виходить з черепа ця гілка? A 54-year-old man was admitted to the neurosurgery department with complaints of lack of sensitivity of the skin of the lower eyelid, the lateral surface of the nose, and the upper lip. During the examination, the doctor diagnosed inflammation of the second branch of the trigeminal nerve Through which hole does this branch leave the skull?

Остистий отвір Prickly Hole

Верхня очноямкова щілина Upper orbital fissure

Рваний отвір Torn hole

Овальний отвір Oval hole

Круглий отвір Round hole

28 / 200
Пацієнт 46-ти років звернувся до лікаря зі скаргами на болі в дрібних суглобах ніг та рук. Суглоби збільшені, мають вигляд потовщених вузлів. У сироватці встановлено підвищений вміст уратів. Це може бути спричинене: A 46-year-old patient turned to the doctor with complaints of pain in the small joints of the legs and arms. The joints are enlarged, have the appearance of thickened nodes. An elevated urate content was found in the serum . This can be caused by:

Порушенням обміну ліпідів Lipid metabolism disorder

Порушенням обміну вуглеводів Carbohydrate metabolism disorder

Порушенням обміну піримідинів Disruption of pyrimidine exchange

Порушенням обміну пуринів Disruption of purine metabolism

Порушенням обміну амінокислот Amino acid metabolism disorder

29 / 200
У хворого з жовтяницею встановлено: підвищення у плазмі крові вмісту загального білірубіну за рахунок непрямого (вільного), в калі та сечі - високий вміст стеркобіліну, рівень прямого (зв’язаного) білірубіну в плазмі крові в межах норми. Про який вид жовтяниці можна думати? In a patient with jaundice, it was established: an increase in the content of total bilirubin in the blood plasma due to indirect (free), in feces and urine - a high content of stercobilin, the level of direct (with bilirubin in the blood plasma is within the normal range. What type of jaundice can you think about?

Гемолітична Hemolytic

Хвороба Жильбера Gilbert's disease

Механічна Mechanical

Паренхіматозна (печінкова) Parenchymatous (hepatic)

Жовтяниця немовлят Infant Jaundice

30 / 200
У хворого з тромбофлебітом нижніх кінцівок раптово після навантаження виникли задишка, різкий біль у грудях, ціаноз, набухання шийних вен. Яке найбільш імовірне порушення кровообігу виникло у хворого? A patient with thrombophlebitis of the lower extremities suddenly developed shortness of breath, sharp pain in the chest, cyanosis, swelling of the neck veins after exercise. What is the most likely circulatory disorder in the patient?

Тромбоемболія ворітної вени Portal vein thromboembolism

Тромбоемболія легеневої артерії Thromboembolism of the pulmonary artery

Тромбоемболія мезентеріальних судин Mesenteric vessel thromboembolism

Тромбоемболія вінцевих судин Thromboembolism of coronary vessels

Тромбоемболія судин головного мозку Thromboembolism of cerebral vessels

31 / 200
При штовханні штанги спортсмен закидає голову назад для максимального підвищення тонусу м’язів-розгиначів верхніх кінцівок. Де розташовані центри рефлексів, які при цьому виникають? When pushing the barbell, the athlete throws his head back to maximize the tone of the extensor muscles of the upper limbs. Where are the centers of reflexes that arise?

Рухова кора Motor cortex

Червоні ядра Red kernels

Спинний мозок Spinal cord

Базальні ганглії Basal ganglia

Ядра Дейтерса Deuters cores

32 / 200
Хворий 65-ти років, що страждає на атеросклероз, госпіталізований до хірургічного відділення з приводу розлитого гнійного перитоніту. Під час операції діагностовано тромбоз брижових артерій. Яка найбільш імовірна причина перитоніту? A 65-year-old patient suffering from atherosclerosis was admitted to the surgical department for diffuse purulent peritonitis. During the operation, thrombosis of the mesenteric arteries was diagnosed. What is the most likely cause peritonitis?

Геморагічний інфаркт Hemorrhagic heart attack

Iшемiчний інфаркт Ischemic heart attack

Ішемія компресійна Compression ischemia

Ішемія ангіоспастична Angiospastic ischemia

Стаз Stage

33 / 200
У крові чоловіка 26-ти років виявлено 18% еритроцитів сферичної, сплощеної, кулястої та остистої форм. Інші еритроцити були у формі двоввігнутих дисків. Як називається таке явище? In the blood of a 26-year-old man, 18% of erythrocytes were found to be spherical, flattened, spherical and spiny. Other erythrocytes were in the form of biconcave discs. What is this phenomenon called?

Патологічний пойкілоцитоз Pathological poikilocytosis

Еритроцитоз Erythrocytosis

Фізіологічний анізоцитоз Physiological anisocytosis

Фізіологічний пойкілоцитоз Physiological poikilocytosis

Патологічний анізоцитоз Pathological anisocytosis

34 / 200
Хвора страждає від болю в ногах та набряків. В ході обстеження хворої на медіальній поверхні стегна спостерігається набряк, збільшення розміру вен, утворення вузлів. З боку якої вени спостерігається патологія? The patient suffers from pain in the legs and swelling. During the examination of the patient on the medial surface of the thigh, there is swelling, an increase in the size of the veins, and the formation of nodes. On which side of the vein is the pathology observed ?

V profunda femoris V profunda femoris

V femoralis V femoralis

V saphena magna V saphena magna

V saphena parva V saphena parva

V tibialis V tibialis

35 / 200
Знешкодження ксенобіотиків (лікарських засобів, епоксидів, ареноксидів, альдегідів, нітропохідних тощо) та ендогенних метаболітів (естрадіолу, простагландинів, лейкотрієнів) відбувається в печінці шляхом їх кон’югації з: Detoxification of xenobiotics (drugs, epoxides, arenoxides, aldehydes, nitroderivatives, etc.) and endogenous metabolites (estradiol, prostaglandins, leukotrienes) occurs in the liver through their conjugation with:

Гліцином Glycinema

Глутатіоном Glutathione

S-Аденозилметіоніном S-Adenosylmethionine

Фосфоаденозином Phosphoadenosine

Аспарагіновою кислотою Aspartic acid

36 / 200
У хворого на гіпертонічну хворобу виявлено високий рівень реніну в крові. Якому з гіпотензивних засобів слід надати перевагу в цьому випадку? A patient with hypertension has a high level of renin in his blood. Which hypotensive agent should be preferred in this case?

Празозин Prazozin

Анаприлін Anaprilin

Ніфедипін Nifedipine

Дихлотіазид Dichlorothiazide

Лізиноприл Lisinopril

37 / 200
На останньому місяці вагітності вміст фібриногену в плазмі крові в 2 рази вище за норму. Яку швидкість осідання еритроцитів слід при цьому очікувати? In the last month of pregnancy, the content of fibrinogen in the blood plasma is 2 times higher than the norm. What should be the rate of erythrocyte sedimentation?

10-15 мм/годину 10-15 mm/hour

3-12 мм/годину 3-12 mm/hour

5-10 мм/годину 5-10 mm/hour

0-5 мм/годину 0-5 mm/hour

40-50 мм/годину 40-50 mm/hour

38 / 200
На розтині у дитини знайдена некротична ангіна, флегмона шиї, гнійний отит, гнійний менінгіт. Ці зміни найбільш характерні для: Necrotic tonsillitis, phlegmon of the neck, purulent otitis, purulent meningitis were found at the autopsy of the child. These changes are most characteristic of:

Токсичної скарлатини Toxic scarlet fever

Отогенного сепсису Otogenic sepsis

Септичної скарлатини Septic scarlet fever

Дифтерії зіву Pharyngeal diphtheria

Менінгококової інфекції Meningococcal infection

39 / 200
У хворого відзначаються періодичні напади серцебиття (пароксизми), сильне потовиділення, напади головного болю. При обстеженні виявлена гіпертензія, гіперглікемія, підвищення основного обміну, тахікардія. При якій патології наднирників спостерігається подібна картина? The patient has periodic attacks of heart palpitations (paroxysms), profuse sweating, and attacks of headache. During the examination, hypertension, hyperglycemia, increased basic metabolism, tachycardia were detected. What pathology is a similar pattern observed in the adrenal glands?

Гіперфункція мозкового шару Hyperfunction of the cerebral layer

Гіпофункція мозкового шару Hypofunction of the cerebral layer

Гіпофункція кори наднирників Hypofunction of the adrenal cortex

Первинний альдостеронізм Primary aldosteronism

Гіперфункція кори наднирників Hyperfunction of the adrenal cortex

40 / 200
На енцефалограмі людини зареєстровано дельта-ритм. У якому стані вона перебуває? A delta rhythm is registered on a person's encephalogram. What state is it in?

Повільного сну Slow Sleep

Засинання Asleep

Активної бадьорості Active cheerfulness

Пасивної бадьорості Passive cheerfulness

Парадоксального сну Paradoxical dream

41 / 200
У людини з нападом бронхоспазму необхідно зменшити вплив блукаючого нерва на гладеньку мускулатуру бронхів. Які мембранні циторецептори доцільно заблокувати для цього? In a person with an attack of bronchospasm, it is necessary to reduce the influence of the vagus nerve on the smooth muscle of the bronchi. Which membrane cytoreceptors should be blocked for this purpose?

α- та β-адренорецептори α- and β-adrenoceptors

β-адренорецептори β-adrenoceptors

М-холінорецептори M-cholinergic receptors

α-адренорецептори α-adrenoceptors

Н-холінорецептори H-cholinergic receptors

42 / 200
Необхідно оцінити рівень збудливості нерва у хворого. Для цього доцільно визначити для нерва наступну величину: It is necessary to assess the level of nerve excitability in the patient. For this, it is advisable to determine the following value for the nerve:

Потенціал спокою Rest potential

Тривалість потенціалу дії Action potential duration

Амплітуда потенціалу дії Action potential amplitude

Порогова сила подразника Threshold stimulus strength

Критичний рівень деполяризації Critical level of depolarization

43 / 200
Хворому з кардіогенним шоком, гіпотензією, ядухою і набряками ввели неглі-козидний кардіотонік. Який саме препарат був введений хворому? A nonglycoside cardiotonic was administered to a patient with cardiogenic shock, hypotension, dysentery, and edema. What drug was administered to the patient?

Добутамін Dobutamine

Кордіамін Cordiamine

Бемегрид Bemegrid

Кофеїн-бензоат натрію Caffeine sodium benzoate

Етимізол Etimizole

44 / 200
Біосинтез пуринового кільця відбувається на рибозо-5-фосфаті шляхом поступового нарощення атомів азоту і вуглецю та замикання кілець. Джерелом рибозофосфату є наступний процес: The biosynthesis of the purine ring occurs on ribose-5-phosphate by the gradual increase of nitrogen and carbon atoms and the closing of the rings. The source of ribosephosphate is the following process:

Глюконеогенез Gluconeogenesis

Гліколіз Glycolysis

Пентозофосфатний цикл Pentose phosphate cycle

Глікогеноліз Glycogenolysis

Гліконеогенез Glyconeogenesis

45 / 200
У жінки з важкою інтоксикацією, спричиненою сепсисом, який і послужив безпосередньою причиною смерті, на розтині виявлене 'тигрове серце'. Який морфогенетичний механізм розвитку переважно лежить в основі даної дистрофії? In a woman with severe intoxication caused by sepsis, which served as the direct cause of death, an autopsy revealed a 'tiger heart'. What morphogenetic mechanism of development mainly underlies this dystrophy?

Спотворений синтез Distorted synthesis

Трансформація Transformation

- -

Декомпозиція Decomposition

Інфільтрація Infiltration

46 / 200
Хворий 32-х років з ураженням ліктьового нерва не може звести до серединної лінії ІІ і V пальці. Функція яких м’язів при цьому порушена? A 32-year-old patient with a lesion of the ulnar nerve cannot bring the 2nd and 5th fingers to the midline. The function of which muscles is impaired?

Долонні міжкісткові м’язи Palmaric interosseous muscles

Тильні міжкісткові м’язи Back interosseous muscles

Короткий долонний м’яз Poralis brevis

Відвідний м’яз мізинця Abductor little finger

Червоподібні м’язи Vermiform muscles

47 / 200
Який з нижченаведених сечогінних засобів слід призначити хворому з первинним гіперальдостеронізмом? Which of the following diuretics should be prescribed to a patient with primary hyperaldosteronism?

Спіронолактон Spironolactone

Маніт Manit

Фуросемід Furosemide

Триамтерен Triamterene

Гіпотіазид Hypotiazide

48 / 200
У мазку з нальоту на мигдаликах хворого з імовірною дифтерією виявлено палички синього кольору з потовщеннями на полюсах. Який метод фарбування мазків було використано? In a smear from a plaque on the tonsils of a patient with suspected diphtheria, blue rods with thickenings at the poles were found. What method of staining the smears was used?

Грама Gram

Буррі Burri

Леффлера Leffler

Гінса Guinsa

Нейссера Neissera

49 / 200
При розтині померлого, який хворів на туберкульоз, у верхній частці правої легені знайдено порожнину розмірами 3х2 см, яка сполучається з бронхом. Стінка порожнини щільна, має три шари: внутрішній -піогенний, середній - шар туберкульозної грануляційної тканини, зовнішній - сполучнотканинний. Який діагноз найбільш імовірний? During the autopsy of the deceased who had tuberculosis, a 3x2 cm cavity was found in the upper part of the right lung, which connects to the bronchus. The wall of the cavity is dense and has three layers: inner - pyogenic, middle - layer of tuberculous granulation tissue, outer - connective tissue. What is the most likely diagnosis?

Гострий осередковий туберкульоз Acute focal tuberculosis

Фіброзно-осередковий туберкульоз Fibrocellular tuberculosis

Туберкульома Tuberculoma

Фіброзно-кавернозний туберкульоз Fibro-cavernous tuberculosis

Гострий кавернозний тубеокульоз Acute cavernous tubeoculosis

50 / 200
Дитина народилася в стані асфіксії. Який препарат необхідно ввести новонародженому для стимуляції дихання? The child was born in a state of asphyxia. What drug should be administered to the newborn to stimulate breathing?

Етимізол Etimizole

Прозерин Prozerin

Празозин Prazozin

Атропін Atropine

Лобелін Lobelin

51 / 200
Хворий похилого віку страждає на хронічний закреп, в основі якого лежить гіпотонія товстого кишківника. Який препарат слід призначити хворому? An elderly patient suffers from chronic constipation, the basis of which is hypotonia of the large intestine. What drug should be prescribed to the patient?

Бісакодил Bisacodyl

Прозерин Prozerin

Ацеклідин Aceclidine

Касторова олія Castor oil

Натрію сульфат Sodium sulfate

52 / 200
При обстеженні в аналізі крові пацієнта виявлено лейкоцитоз, лімфоцитоз, клітини Боткіна-Гумпрехта на тлі анемії. Про яку хворобу слід думати лікарю? When examining the patient's blood, leukocytosis, lymphocytosis, Botkin-Humprecht cells against the background of anemia were detected. What disease should the doctor think about?

Лімфогранулематоз Lymphogranulomatosis

Хронічний лімфолейкоз Chronic lymphocytic leukemia

Інфекційний мононуклеоз Infectious mononucleosis

Мієломна хвороба Myeloma

Гострий мієлолейкоз Acute myelogenous leukemia

53 / 200
У жінки 37-ми років протягом року періодично виникали інфекційні захворювання бактеріального генезу, їх перебіг був вкрай тривалим, ремісії - короткочасними. При обстеженні виявлена гіпогамаглобулінемія. Порушення функції яких клітин може бути прямою її причиною? A 37-year-old woman periodically developed infectious diseases of bacterial origin during the year, their course was extremely long, remissions were short-lived. During the examination, hypogammaglobulinemia was detected. Violations of the functions of cells can be its direct cause?

Нейтрофіли Neutrophils

Фагоцити Phagocytes

Плазматичні клітини Plasma cells

Макрофаги Macrophages

Лімфоцити Lymphocytes

54 / 200
При гістологічному дослідженні щитоподібної залози визначається значна інфільтрація лімфоцитами з утворенням лімфоїдних фолікулів, руйнування паренхіматозних елементів, розростання волокон сполучної тканини. Для якого захворювання характерна така картина? During a histological examination of the thyroid gland, a significant infiltration of lymphocytes with the formation of lymphoid follicles, destruction of parenchymal elements, growth of connective tissue fibers is determined. Which disease is characterized by such a picture?

Паренхіматозний зоб Parenchymal goiter

Ендемічний зоб Endemic goiter

Колоїдний зоб Colloid goiter

Зоб Хашимото Hashimoto's goiter

Дифузний токсичний зоб Diffuse toxic goiter

55 / 200
У чоловіка 50-ти років при обстеженні було виявлено зниження кількості еритроцитів у крові та підвищення рівня вільного гемоглобіну в плазмі крові (гемоглобінемія). КП становив 0,85. Який вид анемії спостерігається у хворого? A 50-year-old man's examination showed a decrease in the number of red blood cells and an increase in the level of free hemoglobin in the blood plasma (hemoglobinemia). CP was 0.85. What type of anemia is observed in the patient?

Набута гемолітична Acquired hemolytic

Гостра постгеморагічна Acute posthemorrhagic

Спадкова гемолітична Hereditary hemolytic

Хронічна постгеморагічна Chronic posthemorrhagic

Анемія внаслідок порушення еритропоезу Anemia due to impaired erythropoiesis

56 / 200
У хворого 68-ми років, який страждає на серцеву недостатність та впродовж тривалого часу приймає препарати наперстянки, з’явилися явища інтоксикації, які швидко нівелювалися застосуванням донатора сульфгідрильних груп унітіолу. Який механізм терапевтичної дії цього засобу? In a 68-year-old patient who suffers from heart failure and has been taking digitalis drugs for a long time, intoxication symptoms appeared, which were quickly eliminated by the use of a donor of sulfhydryl groups unitiol. What is the mechanism of therapeutic action of this agent?

Підвищує енергозабезпечення міокарду Increases the energy supply of the myocardium

Зменшує накопичення іонізованого кальцію Reduces the accumulation of ionized calcium

Реактивує натрій-калієву-АТФ-азу мембран міокардіоцитів Reactivates sodium-potassium-ATP-ase of myocardiocyte membranes

Гальмує вивільнення калію з міокардіоцитів Inhibits release of potassium from myocardiocytes

Сповільнює надходження натрію до міокардіоцитів Slows sodium intake to myocardiocytes

57 / 200
При визначенні мікробного числа повітря у лікарняній палаті виявилося, що воно становить 1500 клітин/м3 . Які групи мікроорганізмів враховувалися при цьому? When determining the microbial count of the air in the hospital ward, it turned out to be 1500 cells/m3. What groups of microorganisms were taken into account?

Всі патогенні та умовно-патогенні бактерії All pathogenic and opportunistic bacteria

Всі бактерії, що виросли на живильному середовищі All bacteria grown on nutrient medium

Стафілококи та гемолітичні стрептококи Staphylococci and hemolytic streptococci

Збудники госпітальних інфекцій Pathogens of hospital infections

Бактерії та віруси - збудники респіраторних інфекцій Bacteria and viruses are the causative agents of respiratory infections

58 / 200
При проведенні операції на тонкій кишці лікар виявив ділянку слизової оболонки, де на фоні колових складок була присутня поздовжня складка. Який відділ тонкої кишки має таку будову? When performing an operation on the small intestine, the doctor found a section of the mucous membrane where a longitudinal fold was present against the background of circular folds. Which part of the small intestine has this structure?

Початковий відділ jejunum Initial section of jejunum

Pars horizontalis duodeni Pars horizontalis duodeni

Pars descendens duodeni Pars descendens duodeni

Дистальний відділ ileum Distal ileum

Pars ascendens duodeni Pars ascendens duodeni

59 / 200
В гістологічному препараті визначається орган, стінку якого утворюють три оболонки. Внутрішня оболонка складається з ендотелія та тонкого підендотеліального шару. Зовнішня оболонка найтовща. Вкажіть, який орган представлено у препараті? In the histological preparation, an organ is identified, the wall of which is formed by three membranes. The inner membrane consists of endothelium and a thin subendothelial layer. The outer membrane is the thickest. Indicate which organ is represented in the preparation ?

Вена Vienna

Сечовід Ureter

Артерія Artery

Матка Uterus

Серце Heart

60 / 200
У хворого нормально забарвлений кал, у складі якого є велика кількість вільних жирних кислот. Причиною цього є порушення: The patient has normally colored stool, which contains a large amount of free fatty acids. The reason for this is a violation:

Секреції ліпаз Secretions of lipases

Гідролізу жирів Hydrolysis of fats

Жовчоутворення Cholelithiasis

Жовчовиділення Bile

Всмоктування жирів Lip suction

61 / 200
У новонародженого малюка педіатр виявив, що отвір крайньої плоті за величиною не перевищує діаметр сечовивідного каналу і голівка статевого члена не може виходити через такий отвір. Як називається цей стан? In a newborn baby, the pediatrician found that the opening of the foreskin does not exceed the diameter of the urethra and the head of the penis cannot exit through such an opening. What is this condition called?

Фімоз Phimosis

Гермафродитизм Hermaphroditism

Епіспадія Epispadia

Гіпоспадія Hypospadia

Парафімоз Paraphimosis

62 / 200
Під час судово-медичної експертизи жінки, яка загинула у автокатастрофі, знайдено ембріон на стадії ранньої гаструли. Назвіть місце його локалізації за умови нормального розвитку: During the forensic medical examination of a woman who died in a car accident, an embryo at the early gastrula stage was found. Name the place of its localization under the condition of normal development:

Черевна порожнина Abdominal cavity

Ампульна частина яйцепроводу Ampoule part of oviduct

Маткова частина яйцепроводу Uterine part of fallopian tube

Стінка матки Wall of uterus

Яєчник Ovary

63 / 200
В основі розвитку імунних і алергічних реакцій організмом застосовуються однакові механізми відповіді імунної системи на антиген. Визначте основну відмінність алергічних реакцій від імунних: In the basis of the development of immune and allergic reactions, the body uses the same mechanisms of the immune system's response to an antigen. Define the main difference between allergic reactions and immune reactions:

Особливість будови антигенів Particularity of the structure of antigens

Розвиток пошкодження тканин Development of tissue damage

Кількість антигену, що потрапляє Number of incoming antigen

Шляхи потрапляння антигенів до організму Paths of entry of antigens into the body

Спадкова схильність Hereditary tendency

64 / 200
У хворої 49-ти років відзначається обмеження довільних рухів у лівих кінцівках. Тонус м’язів у лівих руці та нозі підвищений за спастичним типом, посилені місцеві сухожилкові рефлекси, виявляються патологічні рефлекси. Який найбільш імовірний механізм призвів до розвитку м’язової гіпертонії та гіперрефлексії? A 49-year-old patient has limitation of voluntary movements in the left limbs. The muscle tone in the left arm and leg is increased according to the spastic type, local tendon reflexes are increased, pathological reflexes are detected. What is the most likely mechanism that led to the development of muscle hypertension and hyperreflexia?

Активація збуджувальних впливів з вогнища інсульту Activation of excitatory influences from the stroke center

Активація мотонейронів внаслідок інсульту Motor neuron activation due to stroke

Гальмування мотонейронів кори головного мозку Inhibition of motoneurons of the cerebral cortex

Зниження гальмівних низхідних впливів Decreasing inhibitory downstream effects

Активація синаптичної передачі імпульсів Activation of synaptic impulse transmission

65 / 200
Чоловік 50-ти років хворіє на хронічний бронхіт, скаржиться на задишку під час фізичного навантаження, постійний кашель з відходженням харкотиння. При обстеженні діагностовано ускладнення - емфізема легень. Чим вона зумовлена? A 50-year-old man suffers from chronic bronchitis, complains of shortness of breath during physical exertion, constant cough with expectoration. During the examination, a complication was diagnosed - emphysema of the lungs. What is it conditioned?

Порушення вентиляційно-перфузійного співвідношення в легенях Violation of the ventilation-perfusion ratio in the lungs

Зменшення перфузії легень Decreased lung perfusion

Зниження еластичних властивостей легень Decreasing the elastic properties of the lungs

Зменшення альвеолярної вентиляції Decreased alveolar ventilation

Зменшення розтяжності легень Reduced lung distensibility

66 / 200
У чоловіка 58-ми років, померлого при наростаючих явищах хронічної серцевої недостатності, діагностовано ревматичний гранульоматозний міокардит. Мікроскопічно в міокарді спостерігаються гранульоми, що складаються з макрофагів з гіперхромними ядрами та світлою цитоплазмою, в центрі - осередок некрозу. Який характер має некроз в середині гранульоми? A 58-year-old man, who died due to increasing symptoms of chronic heart failure, was diagnosed with rheumatic granulomatous myocarditis. Microscopically, granulomas consisting of macrophages with hyperchromic nuclei are observed in the myocardium and light cytoplasm, in the center - a center of necrosis. What is the nature of the necrosis in the middle of the granuloma?

Ценкеровський Tsenkerovsky

Колікваційний Collective

Фібриноїдний Fibrinoid

Жировий Fat

Казеозний Caseose

67 / 200
У хворого з довготривалим запаленням слизової оболонки носової порожнини з’явилися симптоми ураження слизової оболонки верхньощелепної пазухи (гайморит). Через яке утворення носової порожнини стало можливим розповсюдження інфекції? A patient with long-term inflammation of the mucous membrane of the nasal cavity developed symptoms of damage to the mucous membrane of the maxillary sinus (sinusitis). What formation of the nasal cavity made it possible for the infection to spread?

Решітчаста лійка Grateful watering can

Клино-решітчаста заглибина Wedge-lattice depression

Решітчасті комірки Grid cells

Клино-піднебінний отвір Cleft palate

Верхньощелепний отвір Maxillary foramen

68 / 200
Хворому з артеріальною гіпертензією було призначено один з антигіпертензивних засобів. Артеріальний тиск нормалізувався, однак хворого почав турбувати постійний сухий кашель. Який з перерахованих препаратів має таку побічну дію? A patient with arterial hypertension was prescribed one of the antihypertensive drugs. Blood pressure normalized, but the patient began to be bothered by a constant dry cough. Which of the listed drugs has such a side effect?

Лізиноприл Lisinopril

Резерпін Reserpin

Ніфедипін Nifedipine

Клофелін Clofelin

Анаприлін Anaprilin

69 / 200
В анотації до препарату вказано, що він містить антигени збудника черевного тифу, адсорбовані на стабілізованих еритроцитах барана. З якою метою використовують цей препарат? In the annotation to the drug, it is indicated that it contains antigens of the causative agent of typhoid, adsorbed on stabilized erythrocytes of a ram. For what purpose is this drug used?

Для виявлення антитіл в реакції Відаля To detect antibodies in the Vidal reaction

Для виявлення антитіл в реакції зв’язування комплементу To detect antibodies in the complement binding reaction

Для серологічної ідентифікації збудника черевного тифу For serological identification of the causative agent of typhoid fever

Для виявлення антитіл в реакції непрямої гемаглютинації To detect antibodies in the indirect hemagglutination reaction

Для виявлення антитіл в реакції гальмування гемаглютинації To detect antibodies in the hemagglutination inhibition reaction

70 / 200
До дерматолога звернувся хворий зі скаргами на появу гнійничків на шкірі обличчя та шиї. При лабораторному аналізі вмісту гнійних фолікул було виявлено рухомі червоподібні паразити. Вкажіть збудника, який викликав це захворювання: A patient came to the dermatologist with complaints about the appearance of pustules on the skin of the face and neck. During the laboratory analysis of the contents of the purulent follicles, mobile worm-like parasites were found. Specify the causative agent that caused this disease:

Блоха людська Human flea

Блощиця ліжкова Bed bug

Залозниця вугрова Pimple gland

Коростяний свербун Scabies itch

Личинка мухи Fly larva

71 / 200
На розтині тіла хлопчика 8-ми років, що хворів на дифтерію зіва та мигдаликів і помер на другий тиждень від початку захворювання, виявлено зміни в міокарді у вигляді дрібновогнищевих некрозів міокардиоцитів, набряку строми з незначною лімфоцитарною інфільтрацією. Діагностуйте вид міокардиту: The autopsy of an 8-year-old boy who had diphtheria of the pharynx and tonsils and died the second week after the onset of the disease revealed changes in the myocardium in the form of small focal necrosis myocardiocytes, stroma edema with slight lymphocytic infiltration. Diagnose the type of myocarditis:

Септичний Septic

Гранулематозний Granulomatous

Альтеративний Alternative

Інтерстиційний Interstitial

Вогнищевий проміжний ексудативний Focal intermediate exudative

72 / 200
У хворого діагностовано септичний ендокардит. Температура тіла протягом 5-ти днів коливалася в межах 39,5°С - 40,2°С. На 6-й день на тлі різкого зниження температури до 35,2°С розвинувся колапс. Який головний механізм колапсу? The patient was diagnosed with septic endocarditis. The body temperature for 5 days fluctuated between 39.5°С - 40.2°С. On the 6th day a collapse developed against the background of a sharp drop in temperature to 35.2°C. What is the main mechanism of the collapse?

Поліурія Polyuria

Вазодилатація Vasodilation

Посилене потовиділення Increased sweating

Гіпервентиляція Hyperventilation

Тахікардія Tachycardia

73 / 200
У пацієнта у результаті тривалого блювання відбувається значна втрата шлункового соку, що є причиною порушення кислотно-лужного стану в організмі. Яка з перерахованих форм порушення кислотно-лужного стану має місце? The patient has a significant loss of gastric juice as a result of prolonged vomiting, which is the cause of acid-base disturbance in the body. Which of the listed forms of acid-base disturbance has place?

Метаболічний ацидоз Metabolic acidosis

Газовий ацидоз Gas acidosis

Газовий алкалоз Gas alkalosis

Негазовий ацидоз Nongaseous acidosis

Негазовий алкалоз Nongaseous alkalosis

74 / 200
Хворий 55-ти років хворіє на хронічний гломерулонефрит протягом 15-ти років. Які зміни складу крові або сечі найбільш характерно свідчать про обмеження секреторної функції нирок? A 55-year-old patient has been suffering from chronic glomerulonephritis for 15 years. What changes in the composition of blood or urine most characteristically indicate a limitation of the secretory function of the kidneys?

Протеїнурія Proteinuria

Гіперазотемія Hyperazotemia

Гіперглікемія Hyperglycemia

Гіпопротеїнемія Hypoproteinemia

Гіпо-, ізостенурія Hypo-, isosthenuria

75 / 200
У хворого в обох щелепах рентгенологічно виявлено численні дефекти у вигляді гладкостінних округлих отворів. При гістологічному дослідженні - явища остеолізису і остеопорозу при явищах слабкого кісткоутворення. В сечі хворого знайдено білок БенсДжонса. Назвіть захворювання: In both jaws of the patient, numerous defects in the form of smooth-walled round holes were detected x-ray. Histological examination revealed the phenomena of osteolysis and osteoporosis with weak bone formation. Protein was found in the patient's urine Bence Jones. Name the disease:

Мієломна хвороба Myeloma

Гострий мієлолейкоз Acute myelogenous leukemia

Гострий недиференційований лейкоз Acute undifferentiated leukemia

Хронічний еритромієлоз Chronic erythromyelosis

Хронічний мієлолейкоз Chronic myelogenous leukemia

76 / 200
Хворий 30-ти років звернувся до лікаря зі скаргами на підвищення температури тіла до 38°С, слабкість, біль у горлі. Об’єктивно: язик вкритий білим нальотом. Які гістологічні структури язика беруть участь в утворенні цього нальоту? A 30-year-old patient turned to the doctor with complaints of an increase in body temperature to 38°C, weakness, sore throat. Objectively: the tongue is covered with a white coating What histological structures of the tongue are involved in the formation of this plaque?

Епітелій ниткоподібних сосочків Epithelium of filiform papillae

Сполучнотканинна основа всіх сосочків язика Connective tissue base of all tongue papillae

Епітелій жолобкуватих сосочків Epithelium of grooved papillae

Епітелій листоподібних сосочків Epithelium of leaf-shaped papillae

Епітелій грибоподібних сосочків Epithelium of mushroom-shaped papillae

77 / 200
Досліджуються рецептори, інформація від яких прямує до кори без участі таламусу. Які це рецептори? Receptors from which information is sent to the cortex without the involvement of the thalamus are being studied. What are these receptors?

Зорові Visual

Смакові Tasty

Слухові Aural

Нюхові Olfactory

Дотикові Touch

78 / 200
На розтині тіла чоловіка похилого віку, який протягом останніх 2-х тижнів страждав на гострий розлад кишківника, виявлені зміни у прямій та сигмоподібній кишках: на поверхні слизової оболонки спостерігається коричнево-зелена плівка. Стінка кишки потовщена, порожнина різко звужена. Мікроскопічно виявляється проникаючий на різну глибину некроз слизової оболонки, некротичні маси пронизані нитками фібрину, з лейкоцитарною інфільтрацією. Який з перерахованих діагнозів найбільш імовірний? At the autopsy of an elderly man who suffered from an acute intestinal disorder for the past 2 weeks, changes were found in the rectum and sigmoid colon: on the surface of the mucous membrane there is brown-green film. The wall of the intestine is thickened, the cavity is sharply narrowed. Microscopically, necrosis of the mucous membrane penetrating to different depths is revealed, necrotic masses are penetrated by fibrin threads, with leukocyte infiltration. Which of the listed diagnoses is the most probable?

Виразковий коліт Ulcerative colitis

Фолікулярний коліт Follicular colitis

- -

Фібринозний коліт Fibrinous colitis

Катаральний коліт Catarrhal colitis

79 / 200
У вагітної жінки взяли кров для підтвердження клінічного діагнозу 'токсоплазмоз'. Яка з перерахованих серологічних реакцій має діагностичне значення? Blood was taken from a pregnant woman to confirm the clinical diagnosis of 'toxoplasmosis'. Which of the listed serological reactions has diagnostic value?

Реакція зв’язування комплементу Complement binding reaction

Реакція нейтралізації Neutralization reaction

Реакція аглютинації Agglutination reaction

Реакція гальмування гемаглютинації Hemagglutination inhibition reaction

Реакція гемадсорбції Hemadsorption reaction

80 / 200
У здорових батьків, спадковість яких не обтяжена, народилася дитина з множинними вадами розвитку. Цитогенетичний аналіз виявив у соматичних клітинах дитини трисомію за 18-ю хромосомою (синдром Едвардса). З яким явищем пов’язане народження такої дитини? A child with multiple developmental disabilities was born to healthy parents whose heredity is not burdened. Cytogenetic analysis revealed trisomy on the 18th chromosome (Edwards syndrome) in the child's somatic cells What phenomenon is associated with the birth of such a child?

Соматичною мутацією у ембріона By somatic mutation in the embryo

Нерозходженням пари хромосом під час гаметогенезу Non-separation of a pair of chromosomes during gametogenesis

Хромосомною мутацією - дуплікацією By chromosomal mutation - duplication

Впливом тератогенних факторів Influence of teratogenic factors

Домінантною мутацією By dominant mutation

81 / 200
Під час об’єктивного обстеження хворого з діагнозом: атеросклеротичний міокардіосклероз, лікар встановив феномен дефіциту пульсу. При якій формі порушення серцевого ритму спостерігається такий феномен? During an objective examination of a patient with a diagnosis of atherosclerotic myocardiosclerosis, the doctor established the phenomenon of a pulse deficiency. In what form of heart rhythm disturbance is this phenomenon observed?

Брадикардія Bradycardia

Миготлива аритмія Atrial fibrillation

Передсердно-шлуночковий ритм Atrial-ventricular rhythm

Синусова екстрасистолія Sinus extrasystole

Ідіовентрикулярний ритм Idioventricular rhythm

82 / 200
При аналізі родоводу пробанда виявлено, що ознака проявляється з однаковою частотою у представників обох статей і хворі наявні у всіх поколіннях (по вертикалі), а по горизонталі - у сибсів (братів і сестер пробанда) з відносно великих родин. Який тип успадкування досліджуваної ознаки? When analyzing the pedigree of the proband, it was found that the trait appears with the same frequency in representatives of both sexes and patients are present in all generations (vertically), and horizontally - in sibs (siblings of the proband) from relatively large families. What is the type of inheritance of the trait under study?

Автосомно-рецесивний Autosomal recessive

Зчеплений з У-хромосомою Linked to Y-chromosome

Зчеплений з Х-хромосомою, рецесивний X-linked recessive

Автосомно-домінантний Autosomal dominant

Зчеплений з Х-хромосомою, домінантний X-linked, dominant

83 / 200
До клініки доставлено хвору на цукровий діабет, госпіталізовано у прекоматозному стані кетоацидотичного типу. Збільшення вмісту якого метаболіту до цього призвело? A diabetic patient was brought to the clinic, hospitalized in a precomatose state of the ketoacidotic type. The increase in the content of which metabolite led to this?

Аспартат Aspartate

α-кетоглутарат α-ketoglutarate

Малонат Malonate

Цитрат Citrate

Ацетоацетат Acetoacetate

84 / 200
В хірургічне відділення доставлено пацієнта з різаною раною медіального краю передпліччя. При обстеженні виявлено, що в хворого перерізано ліктьовий м’яз-згинач зап’ястка і ліктьовий м’яз-розгинач зап’ястка. Які з рухів кисті будуть порушені у хворого? A patient with a cut wound of the medial edge of the forearm was brought to the surgical department. During the examination, it was found that the patient's wrist flexor ulnar muscle and ulnar muscle were cut.' 'Which of the movements of the hand will be impaired in the patient?

Приведення Account

Згинання Bending

Розгинання і відведення Extension and retraction

Відведення Deduction

Розгинання Expansion

85 / 200
При огляді хворого хірург встановив поранення в області двох верхніх третин правої нирки. Цілісність якого органу слід перевірити при цьому, беручи до уваги син-топію правої нирки? When examining the patient, the surgeon found an injury in the area of the upper two thirds of the right kidney. The integrity of which organ should be checked while taking into account the syntopy of the right kidney?

Низхідна ободова кишка Descending colon

Шлунок Stomach

Підшлункова залоза Pancreas

Тонка кишка Small intestine

Печінка Liver

86 / 200
Хворому на гострий інфаркт міокарда у комплексній терапії було призначено гепарин. Через деякий час після введення даного препарату з’явилася гематурія. Який антагоніст гепарину необхідно ввести хворому для усунення даного ускладнення? A patient with an acute myocardial infarction was prescribed heparin in complex therapy. Some time after the administration of this drug, hematuria appeared. What heparin antagonist should be administered to the patient to eliminate this complication?

Вікасол Vikasol

Протаміну сульфат Protamine sulfate

Фібриноген Fibrinogen

Амінокапронова кислота Aminocaproic acid

Неодикумарин Neodicoumarin

87 / 200
У синтезі пуринових нуклеотидів беруть участь деякі амінокислоти, похідні вітамінів, фосфорні ефіри рибози. Коферментна форма якого вітаміну є переносником одновуглецевих фрагментів в цьому синтезі? In the synthesis of purine nucleotides, some amino acids, vitamin derivatives, phosphoric esters of ribose are involved. The coenzyme form of which vitamin is the carrier of one-carbon fragments in this synthesis?

Пантотенова кислота Pantothenic acid

Рибофлавін Riboflavin

Фолієва кислота Folic acid

Піридоксин Pyridoxine

Нікотинова кислота Nicotinic acid

88 / 200
У жінки на шкірі шиї спостерігається новоутворення на ніжці, кулястої форми, зморшкувате. У біоптаті з нього виявлена надмірна проліферація покривного епітелію та строми шкіри у вигляді сосочків, збережена полярність клітин, їх комплексність та базальна мембрана, характерний тканинний атипізм. Який найбільш імовірний діагноз? A woman has a spherical, wrinkled peduncle on the skin of her neck. The biopsy showed excessive proliferation of the covering epithelium and skin stroma in the form of papillae, polarity preserved cells, their complexity and basement membrane, characteristic tissue atypism. What is the most likely diagnosis?

Невус Nevus

Фіброма Fibroma

Рак Cancer

Папілома Papilloma

Саркома Sarcoma

89 / 200
У хворого 75-ти років, який довгий час страждав на атеросклероз церебральних судин, на аутопсії виявлені: тромбоз правої середньої мозкової артерії, великий осередок неправильної форми сірого кашоподібного розм’якшення мозкової тканини. Який патологічний процес розвинувся в спинному мозку? In a 75-year-old patient who suffered from atherosclerosis of cerebral vessels for a long time, the autopsy revealed: thrombosis of the right middle cerebral artery, a large center of an irregular shape of gray mushy size 'improvement of brain tissue. What pathological process has developed in the spinal cord?

Геморагічний інфаркт Hemorrhagic heart attack

Ішемічний інфаркт Ischemic heart attack

- -

Коагуляційний некроз Coagulation necrosis

Гума мозку Brain Gum

90 / 200
У померлої дитини 4-х років при житті була наявна менінгіальна симптоматика, На розтині в м’якій мозковій оболонці макроскопічно виявлені просоподібні вузлики, які мікроскопічно представлені осередком казеозного некрозу з валами епітеліоїдних, лімфоїдних клітин, між якими зустрічаються великі клітини з ядрами на периферії у вигляді півмісяця. Який найбільш імовірний менінгіт у дитини? The deceased 4-year-old child had meningeal symptoms during life. At autopsy, macroscopically, millet-like nodules were found in the meninges, which microscopically represented a focus of caseous necrosis with shafts of epithelioid, lymphoid cells, between which there are large cells with nuclei on the periphery in the form of a crescent. What is the most likely meningitis in a child?

Бруцельозний Brucellosis

Грипозний Influenza

Менінгококовий Meningococcal

Сифілітичний Syphilitic

Туберкульозний Tuberculosis

91 / 200
На препараті представлено орган ендокринної системи, зовні вкритий сполучнотканинною капсулою, від якої всередину органа відходять перегородки, що ділять його на часточки. Кожна часточка складається з двох видів клітин - нейросекреторних пінеалоцитів - полігональних клітин з відростками, локалізованих центрально, та гліоцитів (астроцитів) - на периферії. Що за орган представлено на препараті? The preparation shows an organ of the endocrine system, which is covered on the outside with a connective tissue capsule, from which partitions divide the organ into lobules. Each lobule consists of two types of cells - neurosecretory pinealocytes - polygonal cells with processes located centrally, and gliocytes (astrocytes) - on the periphery. What kind of organ is represented on the preparation?

Епіфіз Pineal gland

Мозкова речовина наднирників Adrenal brain substance

Гіпоталамус Hypothalamus

Гіпофіз Pituitary

Щитоподібна залоза Thyroid

92 / 200
У здорових батьків народилася дитина з синдромом Патау. Який метод медичної генетики дасть змогу віддиференціювати дану спадкову хворобу від її фенокопії? A child with Patau syndrome was born to healthy parents. What method of medical genetics will make it possible to differentiate this hereditary disease from its phenocopy?

Близнюковий Twin

Цитогенетичний Cytogenetic

Дерматогліфічний Dermatoglyphic

Біохімічний Biochemical

Визначення статевого хроматину Definition of sex chromatin

93 / 200
Лікар записав в історії хвороби, що у хворого дихання поверхневе (знижена глибина дихання). Це означає, що зменшеним є такий показник зовнішнього дихання: The doctor recorded in the medical history that the patient's breathing is shallow (reduced depth of breathing). This means that the external breathing rate is reduced:

Життєва ємність легень Vital lung capacity

Ємність вдиху Inhalation capacity

Хвилинний об’єм дихання Minute respiratory volume

Функціональна залишкова ємність Functional residual capacity

Дихальний об’єм Respiratory volume

94 / 200
У хворого 65-ти років діагностовано хворобу Паркінсона. Який засіб, що підвищує вміст дофаміну, слід йому призначити? A 65-year-old patient has been diagnosed with Parkinson's disease. What drug that increases the dopamine content should he be prescribed?

Циклодол Cyclodol

Атропину сульфат Atropine sulfate

Скополаміну гідробромід Scopolamine hydrobromide

Амізил Amizil

Леводопа Levodopa

95 / 200
Після фармакологічної блокади іонних каналів мембрани нервового волокна потенціал спокою зменшився з -90 до -80 м Які канали було заблоковано? After pharmacological blockade of ion channels of the nerve fiber membrane, the resting potential decreased from -90 to -80 m What channels were blocked?

Калієві Potassium

Кальцієві Calcium

Натрієві Sodium

Хлорні Chlorni

Магнієві Magnesium

96 / 200
У студента, який раптово зустрів кохану дівчину, збільшився системний артеріальний тиск. Посилена реалізація яких рефлексів спричинила таку зміну тиску? The student, who suddenly met his girlfriend, had an increase in systemic blood pressure. The increased implementation of which reflexes caused such a change in pressure?

Безумовні симпатичні Unconditional cute

Безумовні парасимпатичні Unconditional parasympathetic

Умовні парасимпатичні Conditional parasympathetic

Умовні симпатичні Conditional cute

Умовні симпатичні та парасимпатичні Conditional sympathetic and parasympathetic

97 / 200
Вагітна жінка під час пологів втратила близько 800 мл крові. Спостерігається тахікардія, артеріальний тиск 100/70 мм рт.ст., тахіпное до 28/хв. Який тип гіпоксії розвивається первинно в такій клінічній ситуації ? A pregnant woman lost about 800 ml of blood during childbirth. Tachycardia is observed, blood pressure is 100/70 mm Hg, tachypnea up to 28/min. What type does hypoxia develop primarily in such a clinical situation?

Змішана Mixed

Дихальна Respiratory

Серцево-судинна Cardiovascular

Тканинна Fabric

Кров’яна Bloody

98 / 200
Основними тригерами, що вмикають ефекторні системи клітини у відповідь на дію гормонів, є протеїнкінази, які змінюють каталітичну активність певних регуляторних ферментів шляхом АТФ-залежного фосфорилювання. Який з наведених ферментів є активним у фосфори-льованій формі? The main triggers that turn on the effector systems of the cell in response to the action of hormones are protein kinases that change the catalytic activity of certain regulatory enzymes through ATP-dependent phosphorylation. Which of the following of enzymes is active in phosphorylated form?

Ілікогенсинтаза Ilicogen synthase

ГОМГ-КоА-редуктаза HOMG-CoA reductase

Ацетил-КоА-карбоксилаза Acetyl CoA carboxylase

Глікогенфосфорилаза Glycogen phosphorylase

Піруваткіназа Pyruvate Kinase

99 / 200
У пацієнта виявлено: поганий апетит, нудота, блювання, анемія. На основі проведеної лабораторної діагностики встановлено дифілоботріоз. Зараження відбулося через вживання: The patient was found to have: poor appetite, nausea, vomiting, anemia. On the basis of the laboratory diagnosis, diphyllobotriosis was established. The infection occurred through the use of:

Свинини Pork

Яловичини Beef

Яєць Egg

Крабів та раків Crabs and crayfish

Риби Fish

100 / 200
Очищення слизової оболонки дихальних шляхів від пилу і мікроорганізмів відбувається завдяки мукоциліарному транспорту - переміщенню слизу поверхнею епітелію. Які клітини забезпечують цей механізм очищення? Cleaning of the mucous membrane of the respiratory tract from dust and microorganisms occurs thanks to mucociliary transport - the movement of mucus across the surface of the epithelium. What cells provide this cleaning mechanism?

Щіточкові Brushes

Бронхіолярні екзокриноцити Bronchiolar exocrinocytes

Війчасті та келихоподібні Ciliate and goblet

Ендокринні та базальні Endocrine and basal

Дендритні Dendritic

101 / 200
На електронномікроскопічній фотографії поперечного зрізу волокна чітко візуалізуються декілька осьових циліндрів з мезаксонами. Яке це волокно? On an electron microscopic photo of a cross-section of a fiber, several axial cylinders with mesaxons are clearly visualized. What kind of fiber is this?

Нервове мієлінове Nerve myelin

Ретикулярне Reticular

Колагенове Collagen

Еластичне Elastic

Нервове безмієлінове Nerve unmyelinated

102 / 200
Інозитолтрифосфати в тканинах організму утворюються в результаті гідролізу фосфатидилінозитолдифосфатів і відіграють роль вторинних посередників (месенджерів) в механізмі дії гормонів. Їхній вплив у клітині спрямований на: Inositol triphosphates in body tissues are formed as a result of hydrolysis of phosphatidylinositol diphosphates and play the role of secondary mediators (messengers) in the mechanism of hormone action. Their influence in the cell is aimed at:

Гальмування фосфодіестерази Inhibition of phosphodiesterase

Вивільнення іонів кальцію з клітинних депо Release of calcium ions from cellular depots

Активацію протеїнкінази А Activation of protein kinase A

Гальмування протеїнкінази С Inhibition of protein kinase C

Активацію аденілатциклази Activation of adenylate cyclase

103 / 200
В ході експерименту було продемонстровано підвищення активності β-галактозидази після внесення лактози до культурального середовища з Е.соlі. Яка ділянка лактозного оперону стає розблокованою від репресору за цих умов? In the course of the experiment, an increase in the activity of β-galactosidase was demonstrated after the addition of lactose to the culture medium with E. coli. Which part of the lactose operon becomes unblocked from the repressor under these conditions?

Оператор Operator

Структурний ген Structural gene

Промотор Promoter

Праймер Primer

Регуляторний ген Regulatory gene

104 / 200
Під час електронномікроскопічного дослідження біоптату гепатоцитів на біліарному полюсі виявлено велику кількість плоских цистерн, сплющених у центральній частині й розширених на периферії, та дрібних міхурців із секреторними гранулами. Назвіть цю структуру: During an electron microscopic examination of a biopsy of hepatocytes at the biliary pole, a large number of flat cisternae, flattened in the central part and expanded at the periphery, and small vesicles with secretory granules were found. Name this structure:

Комплекс Гольджі Golgi Complex

Мікротрубочки Microtubules

Піноцитозні міхурці Pinocytotic vesicles

Ендоплазматична сітка Endoplasmic reticulum

Лізосома Lysosome

105 / 200
При гастробіопсії у хворого встановлена метаплазія поверхневого епітелію слизової оболонки, який замість циліндричного набув вигляду кишкового. Разом з тим спостерігається склероз на місці залоз слизової оболонки та лімфогістіоцитарна інфільтрація. Про яке захворювання шлунка можна думати? During a gastrobiopsy, the patient was diagnosed with metaplasia of the surface epithelium of the mucous membrane, which, instead of being cylindrical, took on the appearance of an intestinal one. At the same time, there was sclerosis at the site of the glands of the mucous membrane and lymphohistiocytic infiltration. About what kind of stomach disease can you think of?

Корозивний гастрит Corrosive gastritis

Хронічний гастрит з ураженням залоз без атрофії Chronic gastritis with gland damage without atrophy

Хронічний атрофічний гастрит Chronic atrophic gastritis

Поверхневий хронічний гастрит Superficial chronic gastritis

Ерозивний гастрит Erosive gastritis

106 / 200
У жінки 23-х років після аборту з’явилася маткова кровотеча. Мікроскопічно у зіскобі з порожнини матки знайдені ворсини хоріона, що нагадують грона винограду. Мікроскопічно: набряк строми ворсин з утворенням безліч кіст, що супроводжуються проліферацією епітелію та синцитію ворсин, залишки плоду і плодових оболонок. Який найбільш імовірний діагноз? A 23-year-old woman developed uterine bleeding after an abortion. Microscopically, chorionic villi resembling bunches of grapes were found in a scraping from the uterine cavity. Microscopically: swelling of the stroma villi with the formation of many cysts, accompanied by the proliferation of the epithelium and syncytium of the villi, remains of the fruit and fruit membranes. What is the most likely diagnosis?

Плацентарний поліп Placental polyp

Хоріонепітеліома Chorioepithelioma

Міхуровий занесок Bubble Drift

Ендометрит Endometritis

Вагітність Pregnancy

107 / 200
У дитячому колективі проведено планову вакцинацію проти кору. Яким методом можна перевірити ефективність проведеної вакцинації? A scheduled vaccination against measles was carried out in the children's team. What method can be used to check the effectiveness of the vaccination?

Вірусологічний Virus

Вірусоскопічний Virusoscopic

Алергопроба Allergy test

Біологічний Biological

Серологічний Serological

108 / 200
Хворому з лихоманкою та висипкою на шкірі після обстеження за допомогою серологічних реакцій поставлено діагноз - фасціольоз. Було встановлено, що хворий заразився шляхом споживання сирої води з річки. Яка стадія життєвого циклу фасціоли інвазійна для людини? A patient with fever and a rash on the skin was diagnosed with fasciolosis after examination using serological reactions. It was established that the patient became infected by consuming raw water from the river. What stage the life cycle of fasciola is invasive for humans?

Метацеркарій Metacercarium

Мірацидій Miracidium

Фіна Fina

Адолескарій Adolescarius

Яйце Egg

109 / 200
Хворий помилково прийняв надмірну дозу тироксину. До яких змін секреції тиреоліберину та тиреотропіну це призведе? The patient mistakenly took an excessive dose of thyroxine. What changes in the secretion of thyroliberin and thyrotropin will this lead to?

Секреція тиреоліберину збільшиться, тиреотропіну - зменшиться Secretion of thyroliberin will increase, thyrotropin will decrease

Секреція тиреотропіну збільшиться, тиреоліберину - зменшиться Secretion of thyrotropin will increase, thyroliberin will decrease

Секреція гормонів зменшиться Hormone secretion will decrease

Змін секреції гормонів не буде There will be no changes in hormone secretion

Секреція гормонів збільшиться Hormone secretion will increase

110 / 200
Турист нещодавно повернувся з країн Середньої Азії, де є багато москітів. У нього на шкірі з’явилися невеликі виразки з нерівними краями. В цьому випадку можна припустити наступне захворювання: The tourist recently returned from the countries of Central Asia, where there are many mosquitoes. He has small sores with uneven edges on his skin. In this case, we can assume the following disease :

Дерматотропний лейшманіоз Dermatotropic leishmaniasis

Демодекоз Demodecosis

Специфічний міаз Specific myiasis

Токсоплазмоз Toxoplasmosis

Скабіес Scabies

111 / 200
У потерпілого з травмою голови у скроневій ділянці діагностовано епідуральну гематому. Яка з артерій найімовірніше пошкоджена? A victim with a head injury in the temporal area was diagnosed with an epidural hematoma. Which of the arteries is most likely damaged?

Середня мозкова Average cerebral

Задня вушна Back auricle

Середня оболонкова Medium shell

Поверхнева скронева Superficial temporal

Передня оболонкова Front Shell

112 / 200
У дитини спостерігається затримка фізичного та розумового розвитку, глибокі порушення з боку сполучної тканини внутрішніх органів, у сечі виявлено кератансульфати. Обмін яких речовин порушений? The child has a delay in physical and mental development, deep disorders of the connective tissue of internal organs, keratan sulfates were detected in the urine. What substances are the metabolism disturbed?

Глікозаміноглікани Glycosaminoglycans

Гіалуронова кислота Hyaluronic acid

Фібронектин Fibronectin

Колаген Collagen

Еластин Elastin

113 / 200
При аналізі ЕКГ виявлено випадіння деяких серцевих циклів PQRST. Наявні зубці та комплекси не змінені. Назвіть вид аритмії: When analyzing the ECG, the loss of some PQRST cardiac cycles was detected. The existing teeth and complexes are not changed. Name the type of arrhythmia:

Синоатріальна блокада Sinoatrial block

Внутрішньопередсердна блокада Intra-atrial blockade

Передсердна екстрасистола Atrial extrasystole

Миготлива аритмія Atrial fibrillation

Атріовентрикулярна блокада Atrioventricular block

114 / 200
До лікарні доставлено хворого з отруєнням інсектицидом - ротеноном. Яка ділянка мітохондріального ланцюга переносу електронів блокується цією речовиною? A patient with rotenone insecticide poisoning was brought to the hospital. What part of the mitochondrial electron transport chain is blocked by this substance?

Цитохром С-оксидаза Cytochrome C oxidase

Сукцинат-коензим О-редуктаза Succinate coenzyme O-reductase

НАДН-коензим О-редуктаза NADH coenzyme O-reductase

Коензим О-цитохром С-редуктаза Coenzyme O-cytochrome C-reductase

АТФ -синтетаза ATP synthetase

115 / 200
В ході регенерації епітелію слизової оболонки порожнини рота (розмноження клітин) відбулася реплікація (авторепродукція) ДНК за напівконсервативним механізмом. При цьому нуклеотиди нової нитки ДНК є комплементарними до: During the regeneration of the epithelium of the mucous membrane of the oral cavity (cell reproduction), replication (autoreproduction) of DNA took place according to a semi-conservative mechanism. At the same time, the nucleotides of the new DNA strand are complementary to:

Інтронних ділянок гену Intron regions of the gene

Ферменту РНК-полімерази RNA polymerase enzyme

Ферменту ДНК-полімерази DNA polymerase enzyme

Змістовних кодонів Content codons

Материнської нитки Mother threads

116 / 200
Серед антиатеросклеротичних препаратів, що застосовуються з метою профілактики та лікування атеросклерозу, є левостатин. Він діє шляхом: Among the antiatherosclerotic drugs used for the prevention and treatment of atherosclerosis is levostatin. It works by:

Активації метаболізму холестерину Activations of cholesterol metabolism

Гальмування біосинтезу холестерину Cholesterol biosynthesis inhibition

Стимулювання екскреції холестерину з організму Stimulation of cholesterol excretion from the body

Усіма наведеними шляхами All given paths

Пригнічення всмоктування холестерину в кишківнику Inhibition of cholesterol absorption in the intestine

117 / 200
При визначенні повітряної та кісткової провідності звуку було встановлено, що у пацієнта ліве вухо краще сприймає звук при кістковому його проведенні, що могло бути пов’язано з захворюванням: When determining the air and bone conduction of sound, it was established that the patient's left ear perceives sound better with bone conduction, which could be related to the disease:

Внутрішнього вуха зліва Left inner ear

Середнього вуха справа Right middle ear

Зовнішнього вуха справа Right external ear

Середнього вуха зліва Left middle ear

Внутрішнього вуха справа Right inner ear

118 / 200
В хронічному експерименті на щурах стимулювали електричним струмом паравентрикулярні та супраоптичні ядра гіпоталамуса. Яка поведінкова реакція спостерігалася у тварин? In a chronic experiment on rats, the paraventricular and supraoptic nuclei of the hypothalamus were stimulated with electric current. What behavioral response was observed in the animals?

Зменшення споживання води Reducing water consumption

Відмова від їжі та рідини Rejection of food and liquids

Збільшення споживання води Increase in water consumption

Збільшення споживання їжі Increase in food consumption

Зменшення споживання їжі Decreasing food intake

119 / 200
У хворого 45-ти років на тлі трансмурального інфаркту міокарда розвинулася гостра лівошлуночкова недостатність. Який лікарський засіб доцільно застосувати у даній ситуації для покращення помпової функції серця? A 45-year-old patient developed acute left ventricular failure against the background of a transmural myocardial infarction. What medicine should be used in this situation to improve the pumping function of the heart?

Еуфілін Euphilin

Добутамін Dobutamine

Ефедрин Ephedrine

Ізадрин Izadrin

Промедол Promedol

120 / 200
При зниженні активності ферментів антиоксидантного захисту посилюються процеси перекисного окиснення ліпідів клітинних мембран. При нестачі якого мікроелементу знижується активність глутатіонпероксидази? When the activity of antioxidant defense enzymes decreases, the processes of peroxidation of lipids in cell membranes increase. In the absence of which trace element, the activity of glutathione peroxidase decreases?

Мідь Copper

Кобальт Cobalt

Селен Selenium

Марганець Manganese

Молібден Molybdenum

121 / 200
У хворого з нагноєнням рани при бактеріологічному дослідженні ранового вмісту виявлено грамнегативну паличку, яка на МПА утворює напівпрозорі слизові колонії синьо-зеленого кольору з перламутровим відтінком. Культура має специфічний запах фіалок або жасмину. Який вид збудника виділений з рани хворого? In a patient with suppuration of a wound, a gram-negative bacillus was found during bacteriological examination of the wound contents, which on MPA forms translucent mucous colonies of blue-green color with a mother-of-pearl shade. The culture has a specific smell violets or jasmine. What type of pathogen was isolated from the patient's wound?

S. pyogenes S. pyogenes

S. faecalis S. faecalis

S. aureus S. aureus

P vulgaris P vulgaris

P aeruginosa P aeruginosa

122 / 200
У хворого під час комп’ютерної томографії грудної клітки діагностовано пухлину заднього нижнього середостіння. Яка з перерахованих структур стиснута пухлиною? The patient was diagnosed with a tumor of the posterior lower mediastinum during a chest CT scan. Which of the listed structures is compressed by the tumor?

N. phrenicus N. phrenicus

Trachea Trachea

Vena cava superior Vena cava superior

Aorta thoracica Aorta thoracica

Arcus aortae Arcus aortae

123 / 200
Під час операції холецистектомії у хірурга виникла необхідність визначити топографію загальної жовчної протоки. Злиттям яких проток утворюється дана анатомічна структура? During the cholecystectomy operation, the surgeon needed to determine the topography of the common bile duct. The fusion of which ducts forms this anatomical structure?

Правої та лівої печінкових проток Right and left hepatic ducts

Загальної печінкової і правої печінкової проток Common hepatic and right hepatic duct

Загальної печінкової і міхурової проток Common hepatic and cystic duct

Загальної печінкової і лівої печінкової проток Common hepatic and left hepatic duct

Лівої печінкової і міхурової проток Left hepatic and cystic duct

124 / 200
Внаслідок ДТП у потерпілої 37-ми років виникло неутримання сечі. Які сегменти спинного мозку пошкоджені? As a result of a road accident, a 37-year-old victim developed urinary incontinence. What segments of the spinal cord are damaged?

Th2 — Th5 Th2 — Th5

S2 - S4 S2 - S4

L1 — L2 L1 — L2

Th1 — Th5 Th1 — Th5

Th1 — L1 Th1 — L1

125 / 200
При обстеженні жінки 56-ти років, що хвора на цукровий діабет 1-го типу, виявлене порушення білкового обміну, що при лабораторному дослідженні крові проявляється аміноацидемією, а клінічно - уповільненням загоєння ран і зменшенням синтезу антитіл. Який з перерахованих механізмів викликає розвиток аміноацидемії? During the examination of a 56-year-old woman with type 1 diabetes, a violation of protein metabolism was detected, which was manifested by aminoacidemia during a laboratory blood test, and clinically - slowing down wound healing and reducing the synthesis of antibodies. Which of the listed mechanisms causes the development of aminoacidemia?

Збільшення ліпопротеїдів високої щільності High-density lipoprotein increase

Зменшення концентрації амінокислот у крові Decreased concentration of amino acids in blood

Підвищення протеолізу Proteolysis increase

Підвищення онкотичного тиску в плазмі крові Increased oncotic pressure in blood plasma

Гіперпротеїнемія Hyperproteinemia

126 / 200
У людини в результаті патологічного процесу збільшена товщина гематоальвеолярного бар’єру. Безпосереднім наслідком цього буде зменшення: The thickness of the blood-alveolar barrier has increased in a person as a result of a pathological process. The direct consequence of this will be a decrease:

Резервного об’єму видиху Exhalation reserve volume

Кисневої ємності крові Blood oxygen capacity

Дифузійної здатності легень Diffusing capacity of lungs

Хвилинного об’єму дихання Minute respiratory volume

Альвеолярної вентиляції легень Alveolar lung ventilation

127 / 200
Недостатність в організмі мікроелементу селену проявляється кардіоміопатією. Імовірною причиною такого стану є зниження активності такого селенвмісного ферменту: Deficiency of the trace element selenium in the body is manifested by cardiomyopathy. The probable cause of this condition is a decrease in the activity of this selenium-containing enzyme:

Цитохромоксидаза Cytochrome oxidase

Лактатдегідрогеназа Lactate dehydrogenase

Глутатіонпероксидаза Glutathione peroxidase

Сукцинатдегідрогеназа Succinate dehydrogenase

Каталаза Catalase

128 / 200
При дослідженні тимуса дитини 5-ти років, що померла від гострої деструктивної стафілококової пневмонії, виявлено зменшення маси залози до 3,0 г. При гістологічному дослідженні в тимусі знайдено: зменшення часточок залози, значне зменшення кількості лімфоцитів з колапсом строми часточок, інверсія шарів, кистоподібне збільшення тілець Гассаля. Який з перерахованих діагнозів найбільш імовірний? When examining the thymus of a 5-year-old child who died of acute destructive staphylococcal pneumonia, a decrease in the mass of the gland to 3.0 g was found. During histological examination in the thymus, : reduction of gland lobules, a significant decrease in the number of lymphocytes with collapse of the stroma of the lobules, inversion of layers, cyst-like enlargement of Hassal's corpuscles. Which of the listed diagnoses is the most probable?

Гіпоплазія тимусу Thymus hypoplasia

Агенезія тимусу Agenesis of the thymus

Дисплазія тимусу Thymic dysplasia

Акцидентальна інволюція тимусу Accidental thymus involution

Тимомегалія Thymomegaly

129 / 200
У хворого внаслідок хронічного обструктивного бронхіту на тлі задишки, тахікардії та ціанозу під час дослідження газового складу крові виявлено розвиток гіпоксемії та гіперкапнії. Яке порушення зовнішнього дихання спостерігається у хворого? The patient developed hypoxemia and hypercapnia as a result of chronic obstructive bronchitis against the background of shortness of breath, tachycardia, and cyanosis during a blood gas composition study. What disturbance of external respiration is observed in the patient?'

Гіповентиляція Hypoventilation

Гіпоперфузія Hypoperfusion

Гіпервентиляція Hyperventilation

Гіперперфузія Hyperperfusion

Гіпердифузія Hyperdiffusion

130 / 200
Хлопчику 5-ти років був встановлений діагноз - міастенія. Оберіть препарат з групи антихолінестеразних засобів, який покращує нервово-м’язову передачу: A 5-year-old boy was diagnosed with myasthenia gravis. Choose a drug from the group of anticholinesterase drugs that improves neuromuscular transmission:

Галантаміну гідробромід Galantamine hydrobromide

Прозерин Proserin

Алоксим Aloxim

Ацеклідин Aceclidine

Армін Armin

131 / 200
Хворому для лікування серцевої недостатності було призначено серцевий глікозид. Яка супутня патологія може сприяти кумуляції серцевих глікозидів? The patient was prescribed a cardiac glycoside for the treatment of heart failure. What concomitant pathology can contribute to the accumulation of cardiac glycosides?

Ниркова недостатність Kidney failure

Гіперацидний гастрит Hyperacid gastritis

Анорексія Anorexia

Гіпертонічна хвороба Hypertensive disease

Гіпоацидний гастрит Hypoacid gastritis

132 / 200
П’ятирічна дитина-правша після черепно-мозкової травми на деякий час втратила здатність розмовляти, але через тривалий час ця здатність у неї відновилась. Яка півкуля була травмована й за рахунок якої властивості ЦНС дітей відновлення мови стало можливим? A five-year-old right-handed child lost the ability to speak for a while after a brain injury, but after a long time this ability was restored. Which hemisphere was injured and due to which properties of the children's central nervous system, the restoration of speech became possible?

Права півкуля, рухливість Right hemisphere, mobility

Обидві півкулі, інертність Both hemispheres, inertia

Права півкуля, пластичність Right hemisphere, plasticity

Ліва півкуля, пластичність Left hemisphere, plasticity

Ліва півкуля, інертність Left hemisphere, inertia

133 / 200
При дослідженні гостроти слуху в коваля виявили втрату слуху на 50% у діапазоні низьких частот і майже нормальну гостроту слуху в діапазоні високих частот. Порушення яких структур слухової системи призвело до такого стану? When examining the hearing acuity of a blacksmith, they found a 50% hearing loss in the low-frequency range and almost normal hearing acuity in the high-frequency range. What structures of the auditory system are impaired in to such a state?

Кортієв орган - ближче до гелікотреми Organ of Corti - closer to helicotrema

Середня частина кортієвого органу Middle part of the organ of Corti

М’язи середнього вуха Middle ear muscles

Барабанна перетинка Eardrum

Кортієв орган - ближче до овального віконця Organ of Corti - closer to the oval window

134 / 200
В підводному човні під час занурення порушилася система подачі кисню. У підводників збільшилися частота дихання і серцевих скорочень. Який вид гіпоксії розвинувся у підводників? The oxygen supply system was broken in the submarine during the dive. The breathing and heart rate of the submariners increased. What type of hypoxia developed in the submariners?

Кров’яна Bloody

Дихальна Respiratory

Серцево-судинна Cardiovascular

Тканинна Fabric

Гіпоксична Hypoxic

135 / 200
Сучасні антиатеросклеротичні препарати застосовуються з метою профілактики та лікування атеросклерозу. Такі препарати як гемфіброзил та фенфібрат гальмують біосинтез холестерину шляхом інгібування ферменту: Modern antiatherosclerotic drugs are used to prevent and treat atherosclerosis. Such drugs as gemfibrozil and fenfibrate inhibit cholesterol biosynthesis by inhibiting the enzyme:

Ацилтрансфераза Acyltransferase

Ілюкозо-6-фосфатаза Ilucose-6-phosphatase

в-ГОМК-редуктаза v-HOMK-reductase

Ацил-КоА-холестеринацилтрансфераза Acyl-CoA-cholesterol acyltransferase

Гексокиназа Hexokinase

136 / 200
Хворий на атеросклероз приймає антисклеротичний засіб - фенофібрат. Який механізм дії має цей засіб? A patient with atherosclerosis takes an antisclerotic drug - fenofibrate. What is the mechanism of action of this drug?

Інгібує абсорбцію холестерину в ШКТ Inhibits absorption of cholesterol in the gastrointestinal tract

Підвищує захоплення ліпопротеїдів низької щільності та блокує біосинтез ендогенного холестерину Increases uptake of low-density lipoproteins and blocks endogenous cholesterol biosynthesis

Поліпшує мікроциркуляцію крові Improves blood microcirculation

Знижує рівень хіломікронів Reduces chylomicrons

Поновлює негативний електричний заряд ендотелію судин Restores the negative electric charge of vascular endothelium

137 / 200
Хворий на гіпертонічну хворобу разом з безсольовою дієтою та з антигіпертензивними засобами, довгий час приймав гідрохлортіазид, що зумовило порушення електролітного балансу. Яке порушення внутрішнього середовища виникло у хворого? A hypertensive patient, along with a salt-free diet and antihypertensive drugs, has been taking hydrochlorothiazide for a long time, which caused a violation of the electrolyte balance. What violation of the patient's internal environment occurred?

Збільшення об’єму циркулюючої крові Increasing the volume of circulating blood

Гіперкаліємія Hyperkalemia

Гіпохлоремічний алкалоз Hypochloremic alkalosis

Гіпермагніємія Hypermagnesemia

Метаболічний ацидоз Metabolic acidosis

138 / 200
При лабораторному дослідженні крові пацієнта виявлено, що вміст білків у плазмі становить 40 г/л. Як це впливає на транскапілярний обмін води в мікроциркуляторному руслі? During the laboratory examination of the patient's blood, it was found that the protein content in the plasma is 40 g/l. How does this affect the transcapillary exchange of water in the microcirculatory channel?

Зменшуються фільтрація і реабсорбція Filtration and reabsorption are decreasing

Збільшується фільтрація, зменшується реабсорбція Filtration increases, reabsorption decreases

Зменшується фільтрація, збільшується реабсорбція Filtration decreases, reabsorption increases

Збільшуються фільтрація і реабсорбція Filtration and reabsorption are increasing

Обмін не змінюється Exchange does not change

139 / 200
При гістологічному дослідженні біоптатів, взятих з потовщених країв виразки шлунка, виявлені невеликі гніздові скупчення різко атипових гіперхромних невеликих епітеліальних клітин, які розташовані серед дуже розвиненої строми. Визначте пухлину: On histological examination of biopsies taken from the thickened edges of a gastric ulcer, small nested clusters of sharply atypical hyperchromic small epithelial cells were found, which are located among a highly developed stroma. Identify the tumor:

Недиференційована саркома Undifferentiated sarcoma

Медулярний рак Medullary cancer

Аденокарцинома Adenocarcinoma

Аденома Adenoma

Скіррозний недиференційований рак Scarcerous undifferentiated cancer

140 / 200
У хворого на ЕКГ виявлено збільшення тривалості комплексу QRS. Наслідком чого це може бути? An increase in the duration of the QRS complex was detected in the patient's ECG. What could be the consequence?

Порушення провідності у атріовентрикулярному вузлі Disruption of conduction in the atrioventricular node

Збільшення часу охоплення збудженням шлуночків Increasing the time of ventricular excitation

Збільшення збудливості шлуночків та передсердь Increased excitability of ventricles and atria

Збільшення збудливості передсердь Increased atrial excitability

Збільшення часу охоплення збудженням передсердь Increasing the time of atrial excitation

141 / 200
В ході авторадіографічного дослідження епітелію тонкої кишки було виявлено, що його повне оновлення відбувається протягом 3-х діб за рахунок проліферації малодиференційованих клітин. Вкажіть їх локалізацію: In the course of an autoradiographic examination of the epithelium of the small intestine, it was found that its complete renewal occurs within 3 days due to the proliferation of poorly differentiated cells. Specify their localization:

Власна пластинка слизової оболонки Own plate of the mucous membrane

Верхівка ворсинок Top of villi

Дно крипт Bottom of Crypts

Основа ворсинок Base of villi

Бічна поверхня ворсинок Lateral surface of villi

142 / 200
У хворого пухлина грудного відділу стравоходу. Куди можуть безпосередньо метастазувати пухлинні клітини? The patient has a tumor of the thoracic esophagus. Where can tumor cells directly metastasize?

Nodi intercostales Nodi intercostales

Nodi mediastinales Nodi mediastinales

Nodi hepatici Nodi hepatici

Ductus thoracicus Ductus thoracicus

Nodi gastrici Nodi gastrici

143 / 200
При загостренні ревматоїдного артриту хворому, в анамнезі якого супутній хронічний гастрит, призначений целекоксиб. Чим обумовлено зменшення побічної дії препарату на травний тракт? For an exacerbation of rheumatoid arthritis, a patient with a history of accompanying chronic gastritis is prescribed celecoxib. What is the reason for the reduction of side effects of the drug on the digestive tract?

Пригнічення фосфоліпази А2 Inhibition of phospholipase A2

Переважне пригнічення циклооксигенази-2 Predominant inhibition of cyclooxygenase-2

Пригнічення фосфодіестерази Inhibition of phosphodiesterase

Переважне пригнічення циклооксигенази-1 Predominant inhibition of cyclooxygenase-1

Переважна стимуляція аденілатциклази Predominant stimulation of adenylate cyclase

144 / 200
Дитина 4-х років госпіталізована в ортопедичне відділення з переломом гомілки зі зсувом. Перед репозицією уламків необхідна анальгезія. Який препарат слід обрати? A 4-year-old child is hospitalized in the orthopedic department with a displaced fracture of the tibia. Analgesia is required before repositioning the fragments. Which drug should be chosen?

Промедол Promedol

Анальгін Analgin

Морфіну гідрохлорид Morphine hydrochloride

Омнопон Omnopon

Панадол Panadol

145 / 200
При напруженій фізичній роботі у м’язовій тканині накопичується молочна кислота, яка дифундує в кров і підхоплюється печінкою та серцем. Який процес забезпечує відновлення запасів глікогену в м’язах? During intense physical work, lactic acid accumulates in muscle tissue, which diffuses into the blood and is picked up by the liver and heart. What process ensures the restoration of glycogen reserves in muscles ?

Цикл лимонної кислоти Cycle of citric acid

Орнітиновий цикл Ornithine cycle

Цикл Корі Cory Cycle

Цикл трикарбонових кислот Cycle of tricarboxylic acids

Пентозофосфатний шлях Pentose phosphate pathway

146 / 200
Фенілкетонурія - це захворювання, яке зумовлено рецесивним геном, що локалізується в аутосомі. Батьки є гетерозиготами за цим геном. Вони вже мають двох хворих синів і одну здорову доньку. Яка імовірність, що четверта дитина, яку вони очікують, народиться теж хворою? Phenylketonuria is a disease caused by an autosomal recessive gene. The parents are heterozygous for this gene. They already have two sick sons and one healthy daughter. What is the probability that the fourth child they are expecting will also be born sick?

100% 100%

25% 25%

0% 0%

75% 75%

50% 50%

147 / 200
Визначення Х-хроматину в соматичних клітинах використовується для експресдіагностики спадкових захворювань, пов’язаних зі зміною кількості статевих хромосом. Який каріотип чоловіка, переважна більшість клітин якого містять одну грудочку Ххроматину: The determination of X-chromatin in somatic cells is used for rapid diagnosis of hereditary diseases associated with changes in the number of sex chromosomes. What is the karyotype of a man whose cells contain a single lump Xchromatin:

46, ХУ 46, ХУ

45, Х0 45, X0

48, ХХХУ 48, XXXXU

47, ХХУ 47, XXU

49, ХХХХУ 49, XXXXX

148 / 200
36-ти років має місце гіповітаміноз В2 148. . Причиною виникнення специфічних симптомів (ураження епітелію, слизових оболонок, шкіри, рогівки ока) імовірно є дефіцит: B2 148 hypovitaminosis has been occurring for 36 years. . The reason for the occurrence of specific symptoms (damages to the epithelium, mucous membranes, skin, cornea of the eye) is probably a deficiency:

Цитохромоксидази Cytochrome oxidases

Цитохрому С Cytochrome C

Флавінових коферментів Flavin coenzymes

Цитохрому В Cytochrome B

Цитохрому А1 Cytochrome A1

149 / 200
На розтині в серці виявлено наступні зміни: великий осередок некрозу білого кольору з червоною облямівкою, який захоплює всю товщу серцевого м’яза. На зовнішній оболонці серця - ознаки фібринозного перикардиту. Який найбільш імовірний діагноз? At autopsy, the following changes were found in the heart: a large center of white necrosis with a red border, which occupies the entire thickness of the heart muscle. On the outer shell of the heart, there are signs of fibrinous pericarditis. What is the most likely diagnosis?

Трансмуральний інфаркт міокарда Transmural myocardial infarction

Субендокардіальний інфаркт міокарда Subendocardial myocardial infarction

Міокардит Myocarditis

Інтрамуральний інфаркт міокарда Intramural myocardial infarction

Субепікардіальний інфаркт міокарда Subepicardial myocardial infarction

150 / 200
В пробірку, що містить розчин NaCl 0,9%, додали краплю крові. Що відбудеться з еритроцитами? A drop of blood was added to a test tube containing a 0.9% NaCl solution. What will happen to the erythrocytes?

Набухання Swelling

Залишаться без змін Remain unchanged

Осмотичний гемоліз Osmotic hemolysis

Зморшкування Wrinkle

Біологічний гемоліз Biological hemolysis

151 / 200
До косметолога звернулася пацієнтка зі скаргами на появу чорних цяток на обличчі. Після обстеження було встановлено, що поява цяток пов’язана з порушенням виділення секрету сальних залоз. Який тип секреції характерний для цих залоз? A patient turned to a cosmetologist with complaints about the appearance of black spots on her face. After the examination, it was established that the appearance of spots is associated with a violation of the secretion of the sebaceous glands. What type secretion characteristic of these glands?

Мерокриновий та мікроапокриновий Merocrine and microapocrine

Голокриновий Holocrine

Макроапокриновий Macroapocrine

Мерокриновий Merocrine

Мікроапокриновий Microapocrine

152 / 200
У хворого 40-ка років ознаки гірської хвороби: запаморочення, задишка, тахікардія, рН крові - 7,50, pCO2 - 30 мм рт.ст., зсув буферних основ +4 ммоль/л. Яке порушення кислотно-основного стану має місце? A 40-year-old patient has signs of mountain sickness: dizziness, shortness of breath, tachycardia, blood pH - 7.50, pCO2 - 30 mm Hg, shift buffer bases +4 mmol/l. What violation of the acid-base state is taking place?

Негазовий ацидоз Nongaseous acidosis

Негазовий алкалоз Nongaseous alkalosis

Видільний ацидоз Excretory acidosis

Газовий ацидоз Gas acidosis

Газовий алкалоз Gas alkalosis

153 / 200
У чоловіка 48-ми років виявлено порушення периферичного кровообігу з обмеженням припливу артеріальної крові, при цьому має місце зблідніння даної ділянки, зниження місцевої температури. Це порушення називається: A 48-year-old man was found to have a violation of peripheral blood circulation with a limitation of arterial blood flow, at the same time there is a pallor of this area, a decrease in local temperature. This violation is called:

Стаз Status

Сладж Sludge

Венозна гіперемія Venous hyperemia

Ішемія Ischemia

Реперфузійний синдром Reperfusion syndrome

154 / 200
Хворому хірург видалив порожнинне утворення печінки діаметром 2 см. Встановлено, що стінка порожнини утворена щільною волокнистою сполучною тканиною, вміст являє собою каламутну, густу, жовтувато-зеленого кольору рідину з неприємним запахом, яка мікроскопічно складається переважно з поліморфноядерних лейкоцитів. Якому патологічному процесу відповідають такі морфологічні зміни? The surgeon removed a 2-cm-diameter cavity of the patient's liver. It was established that the wall of the cavity is formed by dense fibrous connective tissue, the contents are a cloudy, thick, yellowish-green liquid with an unpleasant smell, which microscopically consists mainly of polymorphonuclear leukocytes. What pathological process do such morphological changes correspond to?

Флегмона Phlegmon

Хронічний абсцес Chronic abscess

Емпієма Empyema

- -

Гострий абсцес Acute abscess

155 / 200
Лікар призначив пацієнту з хронічним бронхітом відхаркувальний засіб, який діє шляхом розщеплення дисульфідних зв’язків глікозаміногліканів харкотиння, зменшуючи цим його в’язкість, проте попередив хворого про можливий бронхоспазм при його використанні. Який засіб був призначений? The doctor prescribed an expectorant to a patient with chronic bronchitis, which works by splitting the disulfide bonds of glycosaminoglycans of sputum, thereby reducing its viscosity, but warned the patient about possible bronchospasm when using it. What remedy was prescribed?

Ацетилцистеїн Acetylcysteine

Лібексин Libexin

Трава термопсису Thermopsis grass

Натрію гідрокарбонат Sodium bicarbonate

Бромгексин Bromhexine

156 / 200
Лікар призначив хворому з гострою серцевою недостатністю неглікозидний кардіотонічний засіб, який безпосередньо стимулює β1 -адренорецептори міокарда, що збільшує кровообіг, діурез. Застосовується лише внутрішньовенно крапельно внаслідок швидкої інактивації в організмі. Який препарат призначив лікар? The doctor prescribed a non-glycoside cardiotonic drug to a patient with acute heart failure, which directly stimulates β1-adrenoceptors of the myocardium, which increases blood circulation, diuresis. It is used only intravenously as a drip due to rapid inactivation in organism. What drug did the doctor prescribe?

Дигоксин Digoxin

Адреналін Adrenaline

Анаприлін Anaprilin

Корглікон Corglycon

Добутамін Dobutamine

157 / 200
У молодого подружжя народилася дитина з енцефалопатією. Лікар встановив, що хвороба пов’язана з порушенням мітохондріальної ДНК. Як успадковуються мітохондріальні патології? A young couple had a child with encephalopathy. The doctor established that the disease is associated with a violation of mitochondrial DNA. How are mitochondrial pathologies inherited?

Від обох батьків усіма дітьми From both parents by all children

Від батька тільки дочками From father only through daughters

Від матері тільки синами Only sons from mother

Від батька тільки синами From father only through sons

Від матері всіма дітьми From mother through all children

158 / 200
При розтині тіла жінки, померлої від хронічної ниркової недостатності, в слизовій оболонці товстої кишки виявлені сіро-жовті плівки, що щільно з’єднані з підлеглим шаром, при відокремленні яких утворюються виразки. Мікроскопічно: глибока ділянка некрозу пронизана нитками фібрину. Визначте вид запалення: At the autopsy of the body of a woman who died of chronic kidney failure, gray-yellow films were found in the mucous membrane of the large intestine, tightly connected to the underlying layer, when separated which form ulcers. Microscopically: a deep area of necrosis is penetrated by fibrin threads. Determine the type of inflammation:

Дифтеритичне Diphtheritic

Крупозне True

Гнильне Rotten

Гнійне Purulent

Катаральне Catarrhal

159 / 200
Чоловік 40-ка років перебував у пульмонологічному відділенні з приводу рецидивуючої правосторонньої пневмонії. Помер від легенево-серцевої недостатності. На розтині в правій легені визначається ділянка круглої форми 3х4 см. Вона являє собою порожнину з нерівними шорсткими краями, заповнену каламутною вершкоподібною жовто-зеленою рідиною. Мікроскопічно: стінка порожнини утворена тканиною легені з дифузною інфільтрацією лейкоцитами. Визначте патологічний процес у легені: A 40-year-old man was in the pulmonology department due to recurrent right-sided pneumonia. He died of pulmonary heart failure. An autopsy revealed a round 3x4 cm area in the right lung . It is a cavity with uneven rough edges, filled with a cloudy creamy yellow-green liquid. Microscopically: the wall of the cavity is formed by lung tissue with diffuse infiltration of leukocytes. Define the pathological process in the lung:

Хронічний абсцес Chronic abscess

Емпієма Empyema

Інфаркт Heart attack

Гострий абсцес Acute abscess

Гангрена Gangrene

160 / 200
Під час розтину тіла жінки 52-х років, яка тривалий час хворіла на жовчнокам’яну хворобу, було знайдено: макроскопічно - печінка помірно збільшена, деформована, поверхня органу горбиста, тканина щільна, на розрізі тканина коричнева з зеленим відтінком, складається з множинних вузликів діаметром 8-10 мм. Мікроскопічно - гепатоцелюлярні вузлики оточені прошарками сполучної тканини, яка містить збільшену кількість дрібних жовчних протоків з холестазом. Діагностуйте захворювання печінки: During the autopsy of a 52-year-old woman who had suffered from gallstone disease for a long time, it was found: macroscopically - the liver is moderately enlarged, deformed, the surface of the organ bumpy, the tissue is dense, on cross-section the tissue is brown with a green tint, consists of multiple nodules with a diameter of 8-10 mm. Microscopically, the hepatocellular nodules are surrounded by layers of connective tissue, which contains an increased number of small bile ducts with cholestasis. Diagnose liver disease:

Біліарний цироз печінки Biliary cirrhosis

Портальний цироз печінки Portal cirrhosis

Холелітіаз Cholelithiasis

Токсична дистрофія печінки Toxic liver dystrophy

Постнекротичний цироз печінки Post-necrotic liver cirrhosis

161 / 200
Встановлено, що в клітинах організмів відсутні мембранні органели та їх спадковий матеріал не має нуклеосомної організації. Що це за організми? It was established that the cells of organisms lack membrane organelles and their hereditary material does not have a nucleosome organization. What kind of organisms are these?

Еукаріоти Eukaryotes

Аскоміцети Ascomycetes

Найпростіші The easiest

Віруси Viruses

Прокаріоти Prokaryotes

162 / 200
Аналіз ЕКГ хворого виявив відсутність зубця P. Тривалість та амплітуда QRS комплексу та зубця Т відповідають нормі. Що є водієм ритму серця даного пацієнта? Analysis of the patient's ECG revealed the absence of a P wave. The duration and amplitude of the QRS complex and the T wave correspond to the norm. What is the driver of this patient's heart rhythm?

Міокард шлуночків Myocardium of ventricles

Пучок Гіса His Bundle

Волокна Пуркіньє Purkinje fibers

Передсердно-шлуночковий вузол Atrioventricular node

Синусовий вузол Sine Node

163 / 200
Під час виконання оперативного втручання на гомілці хірург виділяє задню стінку canalis cruropopliteus. Яка анатомічна структура її утворює? During surgery on the lower leg, the surgeon isolates the back wall of the canalis cruropopliteus. What anatomical structure forms it?

M.tibialis anterior M.tibialis anterior

M.gastrocnemius M.gastrocnemius

M.soleus M.soleus

M.plantaris M.plantaris

M.tibialis posterior M.tibialis posterior

164 / 200
Хворій 43-х років для лікування бронхопневмонії призначена бензилпеніциліну натрієва сіль. Який з вказаних побічних ефектів найбільш характерний для даного засобу? A 43-year-old patient was prescribed benzylpenicillin sodium salt for the treatment of bronchopneumonia. Which of the listed side effects is most characteristic of this drug?

Ураження печінки Liver damage

Анемія Anemia

Агранулоцитоз Agranulocytosis

Алергічні реакції Allergic reactions

Неврит слухового нерва Neuritis of the auditory nerve

165 / 200
Після лікування високоефективним протитуберкульозним засобом у жінки 48ми років виникли явища невриту зорового нерва, порушення пам’яті, судоми. Який із зазначених протитуберкульозних препаратів приймала хвора? After treatment with a highly effective antituberculosis agent, a 48-year-old woman developed symptoms of optic neuritis, memory impairment, and convulsions. Which of the indicated antituberculosis drugs did the patient take?

ПАСК PASK

Етамбутол Etambutol

Рифампіцин Rifampicin

Ізоніазид Isoniazid

Канаміцину сульфат Kanamycin sulfate

166 / 200
Хлопчик на другому році життя став часто хворіти на респіраторні захворювання, стоматити, гнійничкові ураження шкіри. Навіть невеликі пошкодження ясен і слизової оболонки ускладнюються запаленням, що протікає тривало. Встановлено, що у крові дитини практично відсутні імуноглобуліни всіх класів. Зниження функціональної активності якої клітинної популяції лежить в основі описаного синдрому? In the second year of his life, the boy began to suffer from respiratory diseases, stomatitis, and pustular skin lesions. Even small damages to the gums and mucous membrane are complicated by long-lasting inflammation. It is established , that there are practically no immunoglobulins of all classes in the child's blood. A decrease in the functional activity of which cell population underlies the described syndrome?

В-лімфоцити B-lymphocytes

Макрофаги Macrophages

NK-лімфоцити NK-lymphocytes

Нейтрофіли Neutrophils

Т-лімфоцити T-lymphocytes

167 / 200
Пацієнт із захворюванням першого верхнього різця зліва скаржиться на сильний біль шкіри в ділянці надбрівної дуги з того ж боку. Реалізація якого виду рефлексів спричиняє вказані реакції? A patient with a disease of the first upper incisor on the left complains of severe skin pain in the area of the suprabrow arch on the same side. Implementation of which type of reflexes causes the indicated reactions?

Сомато-вісцеральні Somato-visceral

Вісцеро-дермальні Viscero-dermal

Пропріоцептивні Proprioceptive

Вісцеро-вісцеральні Viscero-visceral

Вісцеро-соматичні Viscero-somatic

168 / 200
Під час роботи лікарю - стоматологу доводиться довго стояти на ногах, що може призвести до застою крові у венах нижніх кінцівок та їх варикозного розширення. З порушенням якого механізму венозного припливу крові до серця це пов’язано? During work, a doctor - a dentist has to stand on his feet for a long time, which can lead to blood stagnation in the veins of the lower extremities and their varicose expansion. With a violation of which mechanism of venous inflow blood to the heart is it connected?

Присмоктувальний ефект грудної клітки Chest suction effect

Відсутність скорочення скелетних м’язів Absence of skeletal muscle contraction

Залишкова рушійна сила серця Residual cardiac drive

Градієнт тиску Pressure gradient

Присмоктувально-тисковий помповий ефект діафрагми на органи черевної Suction-pressure pumping effect of the diaphragm on abdominal organs

169 / 200
. У хворого 34-х років після перенесеної кишкової інфекції, викликаної сальмонелами, стали згасати симптоми захворювання. Імуноглобуліни якого класу будуть виявлені в крові хворого в період реконвалесценції? . In a 34-year-old patient, after an intestinal infection caused by salmonella, the symptoms of the disease began to disappear. What class of immunoglobulins will be detected in the patient's blood during the period of convalescence?

IgA IgA

IgD IgD

IgM IgM

IgG IgG

IgE IgE

170 / 200
На плановий прийом до педіатра батьки привели дитину віком 13 місяців. Під час повного огляду лікар перевірив розвиток II сигнальної системи дитини. Назвіть період, коли у людини вперше з’являються ознаки розвитку II сигнальної системи: Parents brought a 13-month-old child to a pediatrician for a routine appointment. During a complete examination, the doctor checked the development of the child's II signal system. Name the period when a person first had there are signs of the development of the II signal system:

2,5-3 роки 2.5-3 years

3-5 років 3-5 years

6-12 місяців 6-12 months

2-2,5 роки 2-2.5 years

1,5-2 роки 1.5-2 years

171 / 200
У хворого спостерігається пухлина тканин орбіти позаду очного яблука. Зазначено порушення акомодації та звуження зіниці ока. Яке анатомічне утворення ушкоджено? The patient has a tumor of the orbital tissues behind the eyeball. A violation of accommodation and narrowing of the pupil of the eye is noted. What anatomical formation is damaged?

N. opticus N. opticus

N. trochlearis N. trochlearis

Ganglion ciliare Ganglion ciliare

N. lacrimalis N. lacrimalis

N. nasociliaris N. nasociliaris

172 / 200
До серцево-судинного відділення надійшов хворий зі скаргами на постійний головний біль у потиличній ділянці, шум у вухах, запаморочення. При обстеженні: АТ-180/110 мм рт.ст., ЧСС- 95/хв. Рентгенологічно визначено звуження однієї з ниркових артерій. Активація якої з перерахованих систем викликала гіпертензивний стан хворого? A patient was admitted to the cardiovascular department with complaints of a constant headache in the back of the head, tinnitus, dizziness. During the examination: BP-180/110 mm Hg .art., heart rate - 95/min. Narrowing of one of the renal arteries was determined radiologically. Activation of which of the listed systems caused the hypertensive state of the patient?

Гемостатична Hemostatic

Кінінова Kininova

Симпатоадреналова Sympathoadrenal

Ренін-ангіотензинова Renin-angiotensin

Імунна Immune

173 / 200
Хворого на трансмуральний інфаркт міокарда лівого шлуночка переведено до відділення реанімації у важкому стані. АТ-70/50 мм рт.ст., ЧСС- 56/хв., ЧД- 32/хв. Зазначте головну ланку в патогенезі кардіогенного шоку: A patient with transmural myocardial infarction of the left ventricle was transferred to the intensive care unit in critical condition. BP-70/50 mmHg, heart rate-56/min., ChD- 32/min. Specify the main link in the pathogenesis of cardiogenic shock:

Втрата води Water loss

Падіння периферичного судинного опору Falling of peripheral vascular resistance

Втрата електролітів Electrolyte loss

Крововтрата Blood loss

Падіння серцевого викиду Decreasing cardiac output

174 / 200
У чоловіка 29-ти років з ножовим пораненням шиї спостерігається кровотеча. При первинній обробці рани встановлено, що пошкоджена судина, розташована вздовж латерального краю груднинно-ключично-соскоподібного м’яза. Визначте цю судину: A 29-year-old man with a knife wound to the neck is bleeding. During the initial treatment of the wound, it was established that a vessel located along the lateral edge of the sternoclavicular-mammoid m Identify this vessel:

A. carotis interna A. carotis interna

V jugularis interna V jugularis interna

V jugularis externa V jugularis externa

V jugularis anterior V jugularis anterior

A. carotis externa A. carotis externa

175 / 200
Після перелому нижньої щелепи постраждалий відзначає втрату чутливості шкіри у ділянці підборіддя і нижньої губи. Який нерв був пошкоджений? After a fracture of the lower jaw, the victim notes a loss of skin sensitivity in the area of the chin and lower lip. What nerve was damaged?

Facialis Facialis

Maxillaris Maxillaris

Mentalis Mentalis

Infraorbitalis Infraorbitalis

Buccalis Buccalis

176 / 200
У хворого переливання крові ускладнилося розвитком гемотрансфузійного шоку. Назвіть тип алергічної реакції, що лежить в основі даної патології: The patient's blood transfusion was complicated by the development of hemotransfusion shock. Name the type of allergic reaction underlying this pathology:

Анафілактичний Anaphylactic

Рецептороопосередкований Receptor-mediated

Імунокомплексний Immunocomplex

Гіперчутливість сповільненого типу Delayed type hypersensitivity

Цитотоксичний Cytotoxic

177 / 200
Підшлункова залоза - орган змішаної секреції. Ендокринно продукує бета-клітинами гормон інсулін, який впливає на обмін вуглеводів. Як він впливає на активність глікогенфосфорилази (ГФ) і гліко-генсинтетази (ГС)? The pancreas is an organ of mixed secretion. Endocrinely, beta cells produce the hormone insulin, which affects carbohydrate metabolism. How does it affect the activity of glycogen phosphorylase (GF) and glyco- gene synthetases (GS)?

Не впливає на активність ГФ і ГС Does not affect the activity of HF and HS

Пригнічує ГФ і ГС Suppresses GF and GS

Пригнічує ГФ, активує ГС Suppresses HF, activates HS

Активує ГФ, пригнічує ГС Activates HF, suppresses HS

Активує ГФ і ГС Activates GF and GS

178 / 200
До приймального відділення інфекційної лікарні надійшов чоловік 25-ти років. Діагноз: СНІД. Ураження яких клітин зумовлює стан імунодефіциту? A 25-year-old man was admitted to the reception department of an infectious disease hospital. Diagnosis: AIDS. Damage to which cells causes the state of immunodeficiency?

Т-хелпери T-helpers

Т-кіллери T-killers

Гладкі клітини (тканинні базофіли) Smooth cells (tissue basophils)

Т-супресори T-suppressors

Плазмоцити Plasmocytes

179 / 200
Тривале лікування гіпофункції щитоподібної залози спричинило загальну дистрофію, карієс зубів, тахікардію, тремор кінцівок. Який лікарський засіб викликав зазначені побічні ефекти? Long-term treatment of hypothyroidism caused general dystrophy, dental caries, tachycardia, limb tremors. Which drug caused these side effects?

Тирокальцитонін Tyrocalcitonin

L-тироксин L-thyroxine

Паратиреоїдин Parathyroidin

Хумулін Humulin

Преднізолон Prednisone

180 / 200
Молода людина 25-ти років споживає надмірну кількість вуглеводів (600 г на добу), що перевищує її енергетичні потреби. Який процес буде активуватися в організмі людини у даному випадку? A 25-year-old young man consumes an excessive amount of carbohydrates (600 g per day), which exceeds his energy needs. What process will be activated in the human body in this case ?

Ілюконеогенез Iluconeogenesis

Ліполіз Lipolysis

Гліколіз Glycolysis

Ліпогенез Lipogenesis

Окиснення жирних кислот Oxidation of fatty acids

181 / 200
У пацієнта встановлено гіповітаміноз фолієвої кислоти, що може призвести до порушення синтезу: The patient is diagnosed with hypovitaminosis of folic acid, which can lead to impaired synthesis:

Тимідилових нуклеотидів та жирних кислот Thymidyl nucleotides and fatty acids

Пуринових та тимідилових нуклеотидів Purine and thymidyl nucleotides

Пуринових нуклеотидів та холестерину Purine nucleotides and cholesterol

Цитрату та кетонових тіл Citrate and ketone bodies

Гема та креатину Heme and creatine

182 / 200
У людини виявлена пухлина одного з відділів головного мозку, внаслідок чого в неї порушена здатність підтримувати нормальну температуру тіла. Яка структура головного мозку пошкоджена? A person has a tumor in one of the brain departments, as a result of which his ability to maintain normal body temperature is impaired. What brain structure is damaged?

Таламус Thalamus

Мозочок Cerebellum

Стріатум Striatum

Гіпоталамус Hypothalamus

Чорна субстанція Black substance

183 / 200
До складу харчових раціонів обов’язково входять продукти, в яких є клітковина. Відомо, що вона не перетравлюється ферментами травного тракту й не засвоюється організмом. Яку роль відіграє ця речовина? Food rations must include products that contain fiber. It is known that it is not digested by enzymes of the digestive tract and is not absorbed by the body. What role does this play substance?

Гальмує моторну функцію травного каналу Inhibits the motor function of the alimentary canal

Гальмує процеси виділення ферментів травних соків Inhibits processes of secretion of enzymes of digestive juices

Стимулює моторну функцію травного каналу Stimulates the motor function of the digestive tract

Гальмує секреторну функцію травного каналу Inhibits the secretory function of the alimentary canal

Гальмує всмоктувальну функцію травного каналу Inhibits the absorption function of the alimentary canal

184 / 200
До лікарні звернувся чоловік 50-ти років з розладами пам’яті, болісними відчуттями по ходу нервових стовбурів, зниженням інтелектуальних функцій, порушеннями з боку серцево-судинної системи і явищами диспепсії. В анамнезі хронічний алкоголізм. Дефіцит якого вітаміну може викликати ці симптоми? A 50-year-old man came to the hospital with memory disorders, painful sensations along the course of the nerve trunks, a decrease in intellectual functions, disorders of the cardiovascular system and phenomena of dyspepsia. There is a history of chronic alcoholism. Which vitamin deficiency can cause these symptoms?

Тіамін Thiamine

Ретинол Retinol

Рибофлавін Riboflavin

Ніацин Niacin

Кальциферол Calciferol

185 / 200
По приїзді групи експертів на місце злочину виявлено тіло без ознак життя. В ході дослідження крові загиблого виявлена велика концентрація іонів ціанової кислоти. Інгібування якого комплексу дихального ланцюга мітохондрій стало причиною смерті? Upon the arrival of a group of experts at the scene of the crime, a body was found without signs of life. During the examination of the deceased's blood, a high concentration of cyanic acid ions was found. Inhibition of which mitochondrial respiratory chain complex caused of death?

І And

V V

II II

IV IV

III III

186 / 200
Вивчення організму мешканця Паміру виявило високий рівень основного обміну, розширення грудної клітки, зростання кисневої ємності крові за рахунок збільшення еритроцитів, високий вміст гемоглобіну. До якого адаптивного екологічного типу слід віднести цього чоловіка? Study of the body of a resident of the Pamirs revealed a high level of basic metabolism, expansion of the chest, an increase in the oxygen capacity of the blood due to an increase in erythrocytes, a high content of hemoglobin. To which adaptive ecological type should carry this man?

Гірський Mountain

Арктичний Arctic

Пустельний Desert

Тропічний Tropical

Субтропічний Subtropical

187 / 200
Пацієнт звернувся зі скаргами на гострий біль у правому підребер’ї. При огляді лікар звернув увагу на пожовтіння склер хворого. Лабораторно: підвищена активність АлАТ та негативна реакція на стеркобілін в калі. Для якого захворювання характерні такі симптоми? The patient complained of acute pain in the right hypochondrium. During the examination, the doctor noted the yellowing of the patient's sclera. Laboratory: increased activity of ALT and a negative reaction to stercobilin in feces. What disease is characterized by such symptoms?

Гемолітична жовтяниця Hemolytic Jaundice

Хронічний коліт Chronic Colitis

Гепатит Hepatitis

Хронічний гастродуоденіт Chronic gastroduodenitis

Хронічний гастрит Chronic gastritis

188 / 200
Жінка 30-ти років народила хлопчика з розщепленням верхньої губи ('заяча губа' 'вовча паща'). При додатковому обстеженні виявлені значні порушення нервової, серцевосудинної систем та зору. При дослідженні каріотипу діагностована трисомія за 13-ю хромосомою. Який синдром наявний у хлопчика? A 30-year-old woman gave birth to a boy with a cleft upper lip ('hare's lip', 'wolf's mouth'). Additional examination revealed significant disorders of the nervous, cardiovascular systems and vision. During the karyotype study, trisomy on the 13th chromosome was diagnosed. What syndrome does the boy have?

Клайнфельтера Klinefelter

Дауна Down

Едвардса Edwards

Шерешевського-Тернера Shereshevsky-Turner

Патау Patau

189 / 200
До медико-генетичної консультації звернулася жінка 30-ти років у якої в ядрах більшості клітин епітелію слизової оболонки щоки було виявлено по два тільця Барра. Який попередній діагноз можна встановити? A 30-year-old woman turned to medical and genetic counseling, in which two Barr bodies were detected in the nuclei of most of the cells of the mucous membrane of the cheek. What preliminary diagnosis can be established ?

Моносомія за X-хромосомою Monosomy on the X-chromosome

Трисомія за X-хромосомою Trisomy on X-chromosome

Трисомія за 21-ю хромосомою Trisomy on the 21st chromosome

Трисомія за 18-ю хромосомою Trisomy on the 18th chromosome

Трисомія за 13-ю хромосомою Trisomy on the 13th chromosome

190 / 200
При розтині хворої 28-ми років, що померла від уремії, виявлені збільшені строкаті нирки з осередками крововиливів. Патогістологічно в судинних клубочках виявлені гематоксилінові тільця, капілярні мембрани клубочків у вигляді дротяних петель, гіалінові тромби та осередки фібриноїдного некрозу. За патогенезом гіперчутливість якого типу лежить в основі описаної хвороби? At the autopsy of a 28-year-old patient who died of uremia, enlarged variegated kidneys with foci of hemorrhages were found. Pathohistologically, hematoxylin bodies were found in vascular glomeruli, capillary membranes of glomeruli in in the form of wire loops, hyaline thrombi and foci of fibrinoid necrosis. What type of hypersensitivity is the basis of the described disease in terms of pathogenesis?

Гіперчутливість I типу (анафілактична) Type I hypersensitivity (anaphylactic)

Гіперчутливість III типу (імунокомплексна) Type III hypersensitivity (immunocomplex)

Гіперчутливість II типу (антитілозалежна) Type II hypersensitivity (antibody dependent)

Гіперчутливість IV типу (клітинна цитoтоксичність) Type IV hypersensitivity (cellular cytotoxicity)

Гіперчутливість V типу (гранулематоз) V type hypersensitivity (granulomatosis)

191 / 200
На розтині померлого було виявлено наявність крові в тонкій та товстій кишках (1,5-2 л). В нижніх відрізках клубової кишки стінка мала 'брудні' виразки на місці групових лімфатичних фолікулів. Решта пейєрових бляшок була зеленуватого кольору з демаркаційним запаленням навколо. Було діагностовано черевний тиф. Для якого періоду черевного тифу найбільш характерні описані морфологічні зміни кишки й ускладнення? The autopsy of the deceased revealed the presence of blood in the small and large intestines (1.5-2 l). In the lower segments of the ileum, the wall had 'dirty' ulcers on in the place of group lymphatic follicles. The rest of the Peyer's patches were greenish in color with demarcation inflammation around them. Typhoid fever was diagnosed. For which period of typhoid fever are the described morphological changes of the intestine and complications most characteristic?

Некроз Necrosis

Загоювання виразок Healing Ulcers

Мозкоподібного набухання Cerebral swelling

Чистих виразок Clean ulcers

Утворення виразок Formation of ulcers

192 / 200
Ефективна діагностика носійства збудників кишкових інфекцій ґрунтується на виявленні антитіл до певних антигенів бактерій в реакції непрямої гемаглютинації. Який стандартний препарат слід застосувати у цій реакції? Effective diagnosis of the carriage of pathogens of intestinal infections is based on the detection of antibodies to certain bacterial antigens in the indirect hemagglutination reaction. What standard drug should be used in this reaction?

Антитіла проти імуноглобулінів основних класів Antibodies against immunoglobulins of the main classes

Моноклональні антитіла Monoclonal antibodies

Еритроцитарні діагностикуми з адсорбованими антигенами бактерій Erythrocyte diagnostics with adsorbed bacterial antigens

Монорецепторні діагностичні сироватки Monoreceptor diagnostic sera

Еритроцити барана й гемолітичну сироватку Ram erythrocytes and hemolytic serum

193 / 200
У туриста під час тривалого перебування на спекоті відбулася значна втрата води, що супроводжувалося різким зниженням діурезу. Посилення секреції яких гормонів відбувається при цьому? During a long stay in the heat, a tourist experienced significant water loss, which was accompanied by a sharp decrease in urine output. What hormones are secreted during this?

Вазопресин й альдостерон Vasopressin and aldosterone

Серотонін і дофамін Serotonin and dopamine

Ілюкокортикоїди й інсулін Ilucocorticoids and insulin

Тироксин і трийодтиронін Thyroxin and triiodothyronine

Адреналін і норадреналін Adrenaline and norepinephrine

194 / 200
Після травми на рентгенограмі тазу виявлена тріщина кістки, яка має затульну борозну. Яка це кістка? After the injury, the X-ray of the pelvis revealed a bone crack that has a hidden groove. What bone is it?

Клубова Club

Лобкова Lobkova

Крижова Kryzhova

Сіднична Gluteal

Куприкова Kuprykova

195 / 200
У пацієнта стоматологічного відділення виявлено хворобу Педжета, що супроводжується деградацією колагену. Вирішальним фактом для постановки діагнозу було виявлення у сечі хворого підвищеного рівня: A patient of the dental department was diagnosed with Paget's disease, which is accompanied by collagen degradation. The decisive fact for establishing the diagnosis was the detection of an elevated level in the patient's urine:

Серину Serina

Аланіну Alanine

Триптофану Tryptophan

Оксипроліну Oxyproline

Аргініну Arginine

196 / 200
Школяр 8-ми років звернувся до стоматолога з герпетичним висипанням на нижній губі. Який найбільш ефективний засіб слід призначити? An 8-year-old student went to the dentist with a herpetic rash on the lower lip. What is the most effective remedy to prescribe?

Ампіцилін Ampicillin

Фурадонін Furadonin

Ацикловір Acyclovir

Кетоконазол Ketoconazole

Оксацилін Oxacillin

197 / 200
У пацієнта перед кардіологічною операцією зареєстровано тиск у всіх відділах серця. Який тиск в лівому шлуночку під час діастоли? The pressure in all parts of the heart was recorded in the patient before cardiac surgery. What is the pressure in the left ventricle during diastole?

40 мм рт.ст. 40 mm Hg

0 мм рт.ст. 0 mmHg

120 мм рт.ст. 120 mm Hg

80 мм рт.ст. 80 mm Hg

100 мм рт.ст. 100 mm Hg

198 / 200
Хворий 47-ми років впродовж останніх 3-х років хворіє на туберкульоз легень, скаржиться на задишку, важкість в області правого боку грудної стінки, температуру тіла 37,7°С. Виявлено правобічний ексудативний плеврит. Який тип клітин передбачається у плевральному пунктаті? A 47-year-old patient has been suffering from pulmonary tuberculosis for the past 3 years, complains of shortness of breath, heaviness in the area of the right side of the chest wall, body temperature 37.7 °С. Right-sided exudative pleurisy was detected. What type of cells is expected in the pleural punctate?

Еозинофіли Eosinophils

Еритроцити Erythrocytes

Нейтрофіли Neutrophils

Атипові клітини Atypical cells

Лімфоцити Lymphocytes

199 / 200
У п’ятирічного хлопчика спостерігалися малий зріст, розумове відставання, обмежені рухи, грубі риси обличчя. ЦІ особливості стали помітними з 18-місячного віку. У нього виявили дефіцит L-ідуронідази. Обмін яких сполук порушено? A five-year-old boy had short stature, mental retardation, limited movements, coarse facial features. THESE features became noticeable from the age of 18 months. He was diagnosed with a deficit L-iduronidase. The exchange of which compounds is disturbed?

Білки Proteins

Нуклеотиди Nucleotides

Фосфоліпіди Phospholipids

Вітаміни Vitamins

Глікозаміноглікани Glycosaminoglycans

200 / 200
Дитячий стоматолог мав контакт з хворим на дифтерію підлітком. Час останньої імунізації лікаря проти дифтерії - 12 років. Який препарат необхідно ввести стоматологу? A pediatric dentist had contact with a teenager suffering from diphtheria. The time of the doctor's last immunization against diphtheria is 12 years. What drug should be administered to the dentist?

Інтерферон Interferon

Антитоксична протидифтерійна сироватка Anti-toxic anti-diphtheria serum

Хімічна вакцина Chemical vaccine

Жива вакцина Live vaccine

Рекомбінантна вакцина Recombinant vaccine